Мощность при параллельном и последовательном соединении резисторов: Как определить мощность резисторов. Мощность резисторов при параллельном соединении

Содержание

Как определить мощность резисторов. Мощность резисторов при параллельном соединении

Все электронные устройства содержат резисторы, являющиеся их основным элементом. С его помощью изменяют величину тока в электрической цепи. В статье приведены свойства резисторов и методы расчёта их мощности.

Назначение резистора

Для регулировки тока в электрических цепях применяются резисторы. Это свойство определено законом Ома:

I=U/R (1)

Из формулы (1) хорошо видно, что чем меньше сопротивление, тем сильнее возрастает ток, и наоборот, чем меньше величина R, тем больше ток. Именно это свойство электрического сопротивления используется в электротехнике. На основании этой формулы создаются схемы делителей тока, широко применяющиеся в электротехнических устройствах.

мощность резисторов

В этой схеме ток от источника делится на два, обратно пропорциональных сопротивлениям резисторов.

Кроме регулировки тока, резисторы используются в делителях напряжения. В этом случае опять используется закон Ома, но немного в другой форме:

U=I∙R (2)

Из формулы (2) следует, что при увеличении сопротивления увеличивается напряжение. Это свойство используется для построения схем делителей напряжения.

мощность резисторов на схеме

Из схемы и формулы (2) ясно, что напряжения на резисторах распределяются пропорционально сопротивлениям.

Изображение резисторов на схемах

По стандарту резисторы изображаются прямоугольником с размерами 10 х 4 мм и обозначаются буквой R. Часто указывается мощность резисторов на схеме. Изображение этого показателя выполняется косыми или прямыми чёрточками. Если мощность более 2 Ватт, то обозначение производится римскими цифрами. Обычно это делается для проволочных резисторов. В некоторых государствах, например в США, применяются другие условные обозначения. Для облегчения ремонта и анализа схемы часто приводится мощность резисторов, обозначение которых выполняется по ГОСТ 2.728-74.

Технические характеристики устройств

Основная характеристика резистора – номинальное сопротивление Rн, которое указывается на схеме возле резистора и на его корпусе. Единица измерения сопротивления – ом, килоом и мегаом. Изготавливаются резисторы с сопротивлением от долей ома и до сотен мегаомов. Существует немало технологий производства резисторов, все они имеют и преимущества, и недостатки. В принципе, не существует технологии, которая позволила бы абсолютно точно изготавливать резистор с заданным значением сопротивления.

Второй важной характеристикой является отклонение сопротивления. Оно измеряется в % от номинального R. Существует стандартный ряд отклонения сопротивления: ±20, ±10, ±5, ±2, ±1% и далее вплоть до значения ±0,001%.

Следующей важной характеристикой является мощность резисторов. При работе они нагреваются от проходящего по ним тока. Если рассеиваемая мощность будет превышать допустимое значение, то устройство выйдет из строя.

Резисторы при нагревании изменяют своё сопротивление, поэтому для устройств, работающих в широком диапазоне температур, вводится ещё одна характеристика – температурный коэффициент сопротивления. Он измеряется в ppm/°C, то есть 10-6 Rн/°C (миллионная часть от Rн на 1°C).

Последовательное соединение резисторов

Резисторы могут соединяться тремя разными способами: последовательным, параллельным и смешанным. При последовательном соединении ток поочерёдно проходит через все сопротивления.

как определить мощность резисторов

При таком соединении ток в любой точке цепи один и тот же, его можно определить по закону Ома. Полное сопротивление цепи в этом случае равно сумме сопротивлений:

R=200+100+51+39=390 Ом;

I=U/R=100/390=0,256 А.

Теперь можно определить мощность при последовательном соединении резисторов, она рассчитывается по формуле:

P=I2∙R= 0,2562∙390=25,55 Вт.

Аналогично определяется мощность остальных резисторов:

P1= I2∙R1=0,2562∙200=13,11 Вт;

P2= I2∙R2=0,2562∙100=6,55 Вт;

P3= I2∙R3=0,2562∙51=3,34 Вт;

P4= I2∙R4=0,2562∙39=2,55 Вт.

Если сложить мощность резисторов, то получится полная P:

P=13,11+6,55+3,34+2,55=25,55 Вт.

Параллельное соединение резисторов

При параллельном соединении все начала резисторов подключаются к одному узлу схемы, а концы – к другому. При таком соединении ток разветвляется и течёт по каждому устройству. Величина тока, согласно закону Ома, обратно пропорциональна сопротивлениям, а напряжение на всех резисторах одинаково.

мощность резисторов обозначение

Прежде чем найти ток, нужно рассчитать полную проводимость всех резисторов по общеизвестной формуле:

1/R=1/R1+1/R2+1/R3+1/R4=1/200+1/100+1/51+1/39=0,005+0,01+0,0196+0,0256= 0,06024 1/Ом.

Сопротивление – величина, обратная проводимости:

R=1/0,06024= 16,6 Ом.

Воспользовавшись законом Ома, находят ток через источник:

I= U/R=100∙0,06024=6,024 A.

Зная ток через источник, находят мощность параллельно соединённых резисторов по формуле:

P=I2∙R=6,0242∙16,6=602,3 Вт.

По закону Ома рассчитывается ток через резисторы:

I1=U/R1=100/200=0,5 А;

I2=U/R2=100/100=1 А;

I3=U/R1=100/51=1,96 А;

I1=U/R1=100/39=2,56 А.

Немного по другой формуле можно рассчитать мощность резисторов при параллельном соединении:

P1= U2/R1=1002/200=50 Вт;

P2= U2/R2=1002/100=100 Вт;

P3= U2/R3=1002/51=195,9 Вт;

P4= U2/R4=1002/39=256,4 Вт.

Если всё это сложить, то получится мощность всех резисторов:

P= P1+ P2+ P3+ P4=50+100+195,9+256,4=602,3 Вт.

Смешанное соединение

Схемы со смешанным соединением резисторов содержат последовательное и одновременно параллельное соединение. Эту схему несложно преобразовать, заменив параллельное соединение резисторов последовательным. Для этого заменяют сначала сопротивления R2 и R6 на их общее R2,6, используя формулу, приведённую ниже:

R2,6=R2∙R6/R2+R6.

Точно так же заменяются два параллельных резистора R4, R5 одним R4,5:

R4,5=R4∙R5/R4+R5.

В результате получается новая, более простая схема. Обе схемы приведены ниже.

мощность при последовательном соединении резисторов

Мощность резисторов на схеме со смешанным соединением определяется по формуле:

P=U∙I.

Для расчёта по этой формуле сначала находят напряжение на каждом сопротивлении и величину тока через него. Можно использовать другой метод, чтобы определить мощность резисторов. Для этого используется формула:

P=U∙I=(I∙R)∙I=I2∙R.

Если известно только напряжение на резисторах, то применяют другую формулу:

P=U∙I=U∙(U/R)=U2/R.

Все три формулы часто используются на практике.

Расчёт параметров схемы

Расчёт параметров схемы заключается в нахождении неизвестных токов и напряжений всех ветвей на участках электрической цепи. Имея эти данные, можно рассчитать мощность каждого резистора, входящего в схему. Простые методы расчёта были показаны выше, на практике же дело обстоит сложнее.

В реальных схемах часто встречается соединение резисторов звездой и треугольником, что создаёт значительные трудности при расчётах. Для упрощения таких схем были разработаны методы преобразования звезды в треугольник, и наоборот. Этот метод проиллюстрирован на схеме, представленной ниже:

мощность параллельно соединенных резисторов

Первая схема имеет в своём составе звезду, подключенную к узлам 0-1-3. К узлу 1 подсоединён резистор R1, к узлу 3 – R3, а к узлу 0 – R5. На второй схеме к узлам 1-3-0 подключены резисторы треугольника. К узлу 1 подключены резисторы R1-0 и R1-3, к узлу 3 – R1-3 и R3-0, а к узлу 0 – R3-0 и R1-0. Эти две схемы полностью эквивалентны.

Для перехода от первой схемы ко второй рассчитываются сопротивления резисторов треугольника:

R1-0=R1+R5+R1∙R5/R3;

R1-3=R1+R3+R1∙R3/R5;

R3-0=R3+R5+R3∙R5/R1.

Дальнейшие преобразования сводятся к вычислению параллельно и последовательно соединённых сопротивлений. Когда будет найдено полное сопротивление цепи, находят по закону Ома ток через источник. Используя этот закон, несложно найти токи во всех ветвях.

Как определить мощность резисторов после нахождения всех токов? Для этого используют общеизвестную формулу: P=I2∙R, применяя её для каждого сопротивления, найдём их мощности.

Экспериментальное определение характеристик элементов схемы

Для экспериментального определения нужных характеристик элементов требуется собрать заданную схему из реальных компонентов. После этого с помощью электроизмерительных приборов выполняют все необходимые измерения. Этот метод трудоёмкий и дорогостоящий. Разработчики электрических и электронных устройств для этой цели используют моделирующие программы. С помощью них производятся все необходимые вычисления, и моделируется поведение элементов схемы в различных ситуациях. Только после этого собирается опытный образец технического устройства. Одной из таких распространённых программ является мощная система моделирования Multisim 14.0 фирмы National Instruments.

Как определить мощность резисторов с помощью этой программы? Это можно сделать двумя методами. Первый метод – это измерить ток и напряжение с помощью амперметра и вольтметра. Перемножив результаты измерений, получают искомую мощность.

мощность резисторов при параллельном соединении

Из этой схемы определяем мощность сопротивления R3:

P3=U∙I=1,032∙0,02=0,02064 Вт=20,6 мВт.

Второй метод – это непосредственное измерение мощности при помощи ваттметра.

Ключевые словамощность резисторов

Из этой схемы видно, что мощность сопротивления R3 равна P3=20,8 мВт. Расхождение из-за погрешности в первом методе больше. Точно так же определяются мощности остальных элементов.

Соединение резисторов ⋆ diodov.net

Программирование микроконтроллеров Курсы

Соединение резисторов разными способами позволяет получить необходимую величину сопротивления и мощности рассеивания одного эквивалентного резистора. Всего существует три способы соединения резисторов – последовательное, параллельное и смешанное.

Последовательное соединение резисторов

Последовательное соединение резисторов предполагает использование двух и более радиоэлектронных элемента. Конец предыдущего элемента соединяется с началом последующего и так далее. При последовательном соединении сопротивления и мощности рассеивания всех резисторов складываются.
Рассмотрим следующий пример. Соединим последовательно четыре резистора, каждый имеет R = 1 кОм и мощность рассеивания P = 0,25 Вт.

Последовательное соединение резисторов схема

Rобщ = R1 + R2 + R3 + R4 = 1кОм + 1кОм + 1кОм + 1кОм = 4 кОм.

Pобщ = P1 + P2 + P3 + P4 = 0,25 Вт + 0,25 Вт + 0,25 Вт + 0,25 Вт = 1 Вт.

Таким образом, получается один эквивалентный или общий резистор, имеющий следующие параметры:
Rобщ = 4 кОм; Pобщи = 1 Вт.

Последовательное соединение резисторов мощность рассеивания

В последовательной цепи электрической ток протекает одной и той же величины, поэтому электроны на протяжении всего пути неизбежно наталкиваются на все препятствия в виде сопротивлений. С каждым препятствием уменьшается число свободных зарядов, что приводит к снижению силы электрического тока.

Параллельное соединение резисторов

При параллельном соединении резисторов увеличивается количество путей для перемещения свободных зарядов, то есть электронов, из одного участка пути к другому. Поэтому при параллельном соединении резисторов их суммарное (общее, эквивалентное) сопротивление всегда ниже наименьшего сопротивления из всех резисторов.

Параллельное соединение резисторов схема

Параллельное соединение резисторов

Величина, обратная сопротивлению называется проводимостью. Проводимость измеряется в сименсах [См] и обозначается большей латинской буквой G.

G = 1/R = 1/Ом = См

Поэтому при выполнении различных подсчетов в электрических цепях, имеющих параллельное соединение, пользуются проводимостью.

Если сопротивления всех параллельно соединенных резисторов равны, то для определения общего Rобщ достаточно R одного из них разделить на их общее количество:

Если R1 = R2 = R3 = R4 = R

, то

Rобщ = R/4.

Например, каждый из четырех резисторов имеет R = 10 кОм, тогда

Rобщ = 10 кОм/4 = 2,5 кОм.

Параллельное соединение резисторов мощность рассеивания

Мощности рассеивания суммируются также, как и при последовательном соединении.

Смешанное соединение резисторов

Смешанное соединение резисторов представляет собой комбинации последовательных и параллельных соединений. В принципе любую даже самую сложную электрическую цепь, состоящую из источников питания, конденсаторов, диодов, транзисторов и других радиоэлектронных элементов в конкретный момент времени можно заменить резисторами и источниками напряжения, параметры которых изменяются с каждым последующим моментом времени. Для примера изобразим схему, имеющую несколько соединений.

Смешанное соединение резисторов схема

Общее (эквивалентное) сопротивление находится методом «сворачивания» схемы. Сначала определяется общее сопротивление одного отдельного соединения, затем последующего и так далее.

Смешанное соединение резисторов

Теперь самостоятельно подсчитайте общее сопротивления схемы, приведенной ниже.

Смешанное соединение резисторов

Правильный ответ: 2 ома.

Электроника для начинающих

Еще статьи по данной теме

Расчет мощности при параллельном соединении

Все известные виды проводников обладают определенными свойствами, в том числе и электрическим сопротивлением. Это качество нашло свое применение в резисторах, представляющих собой элементы цепи с точно установленным сопротивлением. Они позволяют выполнять регулировку тока и напряжения с высокой точностью в схемах. Все подобные сопротивления имеют свои индивидуальные качества. Например, мощность при паралл ельном и последовательном соединении резисторов будет различной. Поэтому на практике очень часто используются различные методики расчетов, благодаря которым возможно получение точных результатов.

Свойства и технические характеристики резисторов

Как уже отмечалось, резисторы в электрических цепях и схемах выполняют регулировочную функцию. С этой целью используется закон Ома, выраженный формулой: I = U/R. Таким образом, с уменьшением сопротивления происходит заметное возрастание тока. И, наоборот, чем выше сопротивление, тем меньше ток. Благодаря этому свойству, резисторы нашли широкое применение в электротехнике. На этой основе создаются делители тока, использующиеся в конструкциях электротехнических устройств.

Расчет мощности при параллельном соединении

Помимо функции регулировки тока, резисторы применяются в схемах делителей напряжения. В этом случае закон Ома будет выглядеть несколько иначе: U = I x R. Это означает, что с ростом сопротивления происходит увеличение напряжения. На этом принципе строится вся работа устройств, предназначенных для деления напряжения. Для делителей тока используется паралл ельное соединение резисторов, а для делителей напряжения – последовательное.

На схемах резисторы отображаются в виде прямоугольника, размером 10х4 мм. Для обозначения применяется символ R, который может быть дополнен значением мощности данного элемента. При мощности свыше 2 Вт, обозначение выполняется с помощью римских цифр. Соответствующая надпись наносится на схеме возле значка резистора. Мощность также входит в состав маркировки, нанесенной на корпус элемента. Единицами измерения сопротивления служат ом (1 Ом), килоом (1000 Ом) и мегаом (1000000 Ом). Ассортимент резисторов находится в пределах от долей ома до нескольких сотен мегаом. Современные технологии позволяют изготавливать данные элементы с довольно точными значениями сопротивления.

Расчет мощности при параллельном соединении

Важным параметром резистора считается отклонение сопротивления. Его измерение осуществляется в процентах от номинала. Стандартный ряд отклонений представляет собой значения в виде: +20, +10, +5, +2, +1% и так далее до величины +0,001%.

Большое значение имеет мощность резистора. По каждому из них во время работы проходит электрический ток, вызывающий нагрев. Если допустимое значение рассеиваемой мощности превысит норму, это приведет к выходу из строя резистора. Следует учитывать, что в процессе нагревания происходит изменение сопротивления элемента. Поэтому если устройства работают в широких диапазонах температур, применяется специальная величина, именуемая температурным коэффициентом сопротивления.

Для соединения резисторов в схемах используются три разных способа подключения – паралл ельное, последовательное и смешанное. Каждый способ обладает индивидуальными качествами, что позволяет применять данные элементы в самых разных целях.

Мощность при последовательном соединение

При соединение резисторов последовательно электрический ток по очереди проходит через каждое сопротивление. Значение тока в любой точке цепи будет одинаковым. Данный факт определяется с помощью закона Ома. Если сложить все сопротивления, приведенные на схеме, то получится следующий результат: R = 200+100+51+39 = 390 Ом.

Расчет мощности при параллельном соединении

Учитывая напряжение в цепи, равное 100 В, по закону Ома сила тока будет составлять I = U/R = 100/390 = 0,256 A. На основании полученных данных можно рассчитать мощность резисторов при последовательном соединении по следующей формуле: P = I 2 x R = 0,256 2 x 390 = 25,55 Вт.

Таким же образом можно рассчитать мощность каждого отдельно взятого резистора:

  • P1 = I 2 x R1 = 0,256 2 x 200 = 13,11 Вт;
  • P2 = I 2 x R2 = 0,256 2 x 100 = 6,55 Вт;
  • P3 = I 2 x R3 = 0,256 2 x 51 = 3,34 Вт;
  • P4 = I 2 x R4 = 0,256 2 x 39 = 2,55 Вт.

Если сложить полученные мощность, то полная Р составит: Р = 13,11+6,55+3,34+2,55 = 25,55 Вт.

Мощность при паралл ельном соединение

При паралл ельном подключении все начала резисторов соединяются с одним узлом схемы, а концы – с другим. В этом случае происходит разветвление тока, и он начинает протекать по каждому элементу. В соответствии с законом Ома, сила тока будет обратно пропорциональна всем подключенным сопротивлениям, а значение напряжения на всех резисторах будет одним и тем же.

Расчет мощности при параллельном соединении

Прежде чем вычислять силу тока, необходимо выполнить расчет полной проводимости всех резисторов, применяя следующую формулу:

  • 1/R = 1/R1+1/R2+1/R3+1/R4 = 1/200+1/100+1/51+1/39 = 0,005+0,01+0,0196+0,0256 = 0,06024 1/Ом.
  • Поскольку сопротивление является величиной, обратно пропорциональной проводимости, его значение составит: R = 1/0,06024 = 16,6 Ом.
  • Используя значение напряжения в 100 В, по закону Ома рассчитывается сила тока: I = U/R = 100 x 0,06024 = 6,024 A.
  • Зная силу тока, мощность резисторов, соединенных паралл ельно, определяется следующим образом: P = I 2 x R = 6,024 2 x 16,6 = 602,3 Вт.
  • Расчет силы тока для каждого резистора выполняется по формулам: I1 = U/R1 = 100/200 = 0,5A; I2 = U/R2 = 100/100 = 1A; I3 = U/R3 = 100/51 = 1,96A; I4 = U/R4 = 100/39 = 2,56A. На примере этих сопротивлений прослеживается закономерность, что с уменьшением сопротивления, сила тока увеличивается.

Существует еще одна формула, позволяющая рассчитать мощность при паралл ельном подключении резисторов: P1 = U 2 /R1 = 100 2 /200 = 50 Вт; P2 = U 2 /R2 = 100 2 /100 = 100 Вт; P3 = U 2 /R3 = 100 2 /51 = 195,9 Вт; P4 = U 2 /R4 = 100 2 /39 = 256,4 Вт. Сложив мощности отдельных резисторов, получится их общая мощность: Р = Р1234 = 50+100+195,9+256,4 = 602,3 Вт.

Таким образом, мощность при последовательном и паралл ельном соединении резисторов определяется разными способами, с помощью которых можно получить максимально точные результаты.

Расчет мощности при параллельном соединенииПри проектировании электрических схем возникает необходимость использования последовательного и параллельного соединений резисторов. Соединения применяются также и при ремонтах электрооборудования, поскольку в некоторых ситуациях невозможно найти эквивалентный номинал резистора. Выполнить расчет просто, и справиться с этой операцией может каждый.

Типы проводников

Расчет мощности при параллельном соединенииПроводимость веществом электрического тока связана с наличием в нем свободных носителей заряда. Их количество определяется по электронной конфигурации. Для этого необходима химическая формула вещества, при помощи которой можно вычислить их общее число. Значение для каждого элемента берется из периодической системы Дмитрия Ивановича Менделеева.

Электрический ток — упорядоченное движение свободных носителей заряда, на которые воздействует электромагнитное поле. При протекании тока по веществу происходит взаимодействие потока заряженных частиц с узлами кристаллической решетки, при этом часть кинетической энергии частицы превращается в тепловую энергию. Иными словами, частица «ударяется» об атом, а затем снова продолжает движение, набирая скорость под действием электромагнитного поля.

Процесс взаимодействия частиц с узлами кристаллической решетки называется электрической проводимостью или сопротивлением материала. Единицей измерения является Ом, а определить его можно при помощи омметра или расчитать. Согласно свойству проводимости, вещества можно разделить на 3 группы:

  1. Проводники (все металлы, ионизированный газ и электролитические растворы).
  2. Полупроводники (Si, Ge, GaAs, InP и InSb).
  3. Непроводники (диэлектрики или изоляторы).

Расчет мощности при параллельном соединении

Проводники всегда проводят электрический ток, поскольку содержат в своем атомарном строении свободные электроны, анионы, катионы и ионы. Полупроводники проводят электричество только при определенных условиях, которые влияют на наличие или отсутствие свободных электронов и дырок. К факторам, влияющим на проводимость, относятся следующие: температура, освещенность и т. д. Диэлектрики вообще не проводят электричество, поскольку в их структуре вообще отсутствуют свободные носители заряда. При выполнении расчетов каждый радиолюбитель должен знать зависимость сопротивления от некоторых физических величин.

Зависимость сопротивления

Значение электропроводимости зависит от нескольких факторов, которые необходимо учитывать при расчетах, изготовлении элементов резистивной нагрузки (резисторов), ремонте и проектировании устройств. К этим факторам необходимо отнести следующие:

  1. Температура окружающей среды и материала.
  2. Электрические величины.
  3. Геометрические свойства вещества.
  4. Тип материала, из которого изготовлен проводник (полупроводник).

К электрическим величинам можно отнести разность потенциалов (напряжение), электродвижущую силу (ЭДС) и силу тока. Геометрией проводника является его длина и площадь поперечного сечения.

Э

Формулы параллельного, последовательного и смешанного соединения резисторов

Автор Aluarius На чтение 7 мин. Просмотров 568 Опубликовано

Ни одна электрическая схема не обходится без резисторов. Что это такое, для чего он нужен и какими способами их подключают в электрическую цепь рассмотрим подробно.

Что такое резистор и для чего он нужен

Резистор – пассивный элемент электрической цепи, который поглощает энергию тока и преобразовывает её в тепло за счет сопротивления потоку электронов в цепи.

Зависимость тока от сопротивления описывается законом Ома и рассчитывается по формуле I = U/R.

Свойство резисторов ограничивать ток и снижать напряжение используется во многих электронных устройствах и бытовых приборах.

Справка: Резисторы бывают двух видов – постоянные и переменные, во втором случае сопротивление проводника изменяется механическим путем (вручную).

Последовательное и параллельное соединение резисторов – основные способы соединения резистивных элементов.

Внимание! Резистор не имеет полярности, длина выводов с обоих концов одинакова, поэтому для лучшего понимания сути соединения предлагается называть выводы:

  1. С правого края – правый.
  2. С левого края – левый.

Понятие параллельного подключения резисторов

При параллельном подключении правые выводы всех резисторов соединяются в один узел, левые – во второй узел.

паралельное-соединение-резисторов

 

При параллельном включении резисторов ток в цепь разветвляется по отдельным ветвям, протекая через каждый элемент – по закону Ома величина тока обратно пропорциональна сопротивлению, напряжение на всех элементах одинаковое.

соединение-резисторов

 

Справка: Ветвь – фрагмент электрической цепи, содержащий один или несколько последовательно соединенных компонентов от узла до узла.

Последовательное подключение

При последовательном соединении резисторы нужно подключить в цепь друг за другом – правый вывод одного резистора к левому второго, правый второго – к левому третьего и так далее в зависимости от количества соединяемых элементов.

Последовательное подключение резисторов

 

При последовательном соединении ток, не изменяя своей величины, течет через все резистивные элементы.

Последовательное-подключение-резисторов

 

Смешанное подключение

При смешанном подключении в одной схеме сочетаются несколько видов соединений – последовательное, параллельное соединение резисторов и их комбинации. Самую сложную электрическую схему, состоящую из источников питания, диодов, транзисторов, конденсаторов и других радиоэлектронных элементов можно заменить резисторами и источниками напряжения, параметры которых изменяются в каждый момент времени. О параллельном соединении резистора и конденсатора читайте тут.

Смешанное подключение-резисторовСмешанная схема делится на фрагменты, ток и напряжение рассчитывается для каждого отдельно в зависимости от того, как они соединены на выбранном сегменте электрической схемы.

Важно! Для расчета сопротивления резистора в схеме применяют отдельные формулы для каждого конкретного элемента в зависимости от вида соединения.

Что ещё нужно учитывать при подключении резисторов

Важный показатель в работе резистивного элемента мощность рассеивания – переход электрической энергии в тепловую, вызывающую нагрев элемента.

При превышении допустимой мощности рассеивания резисторы будут сильно греться и могут сгореть, поэтому при расчете схем соединения надо учитывать этот параметр – важно знать насколько изменится мощность резистивных элементов при включении в электрическую цепь.

Какая мощность тока при последовательном и параллельном соединении

Определение мощности отдельного резистивного элемента производится по формуле

P = U²/R или P = I²R , которую можно вывести из формулы расчета мощности электрической цепи P = UI по закону Ома.

Мощность при параллельном соединении

Рассчитав сопротивление каждого элемента в отдельности, считаем мощность каждого по формуле P = I²R, где

  • R – не номинальное сопротивление резистивного элемента, а рассчитанное для данной цепи;
  • I – сила тока в цепи.

При параллельном соединении через меньший резистор протекает больший ток – мощность рассеивания на этом резистивном элементе будет больше, чем на остальных.

Важно! При расчете параллельной цепи следует учитывать мощность сопротивления с самым маленьким номиналом.

Мощность при последовательном соединении

Вычислив сопротивление каждого резистивного элемента по отдельности, рассчитываем мощность каждого по формуле P = U²/R, где

  • R – рассчитанное нами сопротивление для определенной схемы;
  • U – падение напряжения на данном резистивном элементе.

Справка: Полную мощность цепи при последовательном и параллельном соединении можно найти, сложив вычисленные мощности отдельных элементов, входящих в цепь Pобщ = P1+P2+P3+…+Pn.

Как правильно рассчитать сопротивление

Применяется закон Ома для участка цепи – расчет сопротивления делается по формуле R = U/I, где

  • U – падение напряжение на конкретном резистивном элементе;
  • I – ток, протекающий через него.

При последовательном соединении

Для двух элементов считаем Rобщ = R1+R2.

Для нескольких сопротивлений разного номинала Rобщ = R1+R2+R3+…+Rn.

При параллельном соединении

Расчет для двух резисторов делаем по формуле Rобщ = (R1×R2)/(R1+R2).

Сопротивление параллельных резисторов с разным номиналом рассчитываем по формуле

Rобщ = 1/(1/R1+1/R2+1/R3+…+1/Rn).

Для элементов, соединенных в параллель, суммарное сопротивление всегда ниже наименьшего номинального.

Как рассчитать сложные схемы соединения резисторов

Сложные схемы рассчитываются путем группировки по параллельному и последовательному способу соединения.

Смешанное подключение-резисторовПеред нами сложная схема – задача рассчитать общее сопротивление:

  1. R2, R3, R4 объединим в последовательную группу – применим формулу R2,3,4 = R2+R3+R4.
  2. R5 и R2,3,4 – параллельно соединенные резисторы, рассчитаем R5,2,3,4 = 1/ (1/R5+1/R2,3,4).
  3. R5,2,3,4, R1, R6 опять объединяем в последовательную группу – суммируя величины, получаем Rобщ = R5,2,3,4+R1+R6.

Для больших схем существуют специальные методы, облегчающие расчет. Один из таких методов – эквивалентное преобразование «треугольника» в «звезду». Такая система расчета применяется в том случае, когда невозможно по схеме определить последовательное или параллельное подключение резисторов.

Преобразование «звезда-треугольник»

Для соединения резистивных элементов, кроме вышеописанных способов, существует несколько других видов соединения:

  • «звезда» – соединение трех ветвей с одним общим узлом;
  • «треугольник» – соединение ветвей схемы в виде треугольника, сторонами которого служат ветви, вершины представляют узлы.

Справка: Узел – точка, в которой соединяются три и более проводника электрической цепи.

Эквивалентность замены предполагает стабильность токов, входящих в каждый узел, при одинаковых напряжения между одноименными узлами «треугольника» и «звезды».

Сопротивление резистора луча «звезды»

Сопротивление резистора луча «звезды» равно произведению сопротивлений резисторов прилегающих сторон «треугольника», деленному на сумму сопротивлений резисторов трех сторон «треугольника».

RA = RAB RAC/(RAB+RAC+RDC).

Сопротивление резисторов сторон «треугольника» равно сумме произведения сопротивлений резисторов двух прилегающих лучей «звезды», деленного на сопротивление третьего луча.

RAB=(RARB+RARC+RBRС)/RC

формулы рассчета звезды резисторов

О разнице подключения звезда и треугольник читайте здесь.

Чему равна сила тока в цепи при параллельном соединении резисторов

Согласно правилу Кирхгофа ток, поступающий в узел, равен току, выходящему из узла, – величина тока до группы параллельных резисторов и после нее должна быть неизменной.

Ток в группе параллельных резисторов распределяется по цепи в зависимости от их номинала, после прохождения через сопротивления суммируется в узле и выходит из него неизменным I = I1+I2+I3+…+In.

Как определить величину эквивалентного сопротивления при последовательном соединении резисторов

Справка: Эквивалентом сопротивления называется замена части схемы, состоящей из нескольких резистивных элементов, одним элементом.

Для последовательного соединения эквивалентное сопротивление равно сумме сопротивлений резисторов, включенных в группу, для расчета применяется формула Rэкв = R1+R2+…+Rn.

Например: Нужно посчитать эквивалентное сопротивление данной схемы.

Смешанное подключение-резисторовРешение задачи производится путем разделения резистивных элементов на системные группы.

Выделяем первую группу из последовательно соединенных элементов – R2, R3, R4.

Считаем сопротивление Rобщ1 = R2+R3+R4.

сложная-схема-подключения-резисторов

Выделяем вторую группу из последовательных элементов R1, R5, R6.

сложная_схема_подключения_резисторов

Считаем сопротивление Rобщ2 = R1+R5+R6.

Получаем величину двух эквивалентных сопротивлений Rобщ1 и Rобщ2, соединенных параллельно.

Делаем расчет всей схемы Rэкв= Rобщ1× Rобш2/ (Rобщ1+ Rобщ2).

Зная способы соединения и формулы расчета можно рассчитать любую сложную схему соединения резистивных элементов, однако существует множество онлайн калькуляторов, которые сделают это быстрей человека, достаточно только ввести нужные параметры компонентов схемы.

Последовательное и параллельное соединение резисторов


Последовательное соединение резисторов

Последовательное соединение резисторов применяется для увеличения сопротивления. Т.е. когда резисторы соединены последовательно, общее сопротивление равняется сумме сопротивлений каждого резистора. Например, если резисторы R1 и R2 соединены последовательно, их общее сопротивление высчитывается по формуле:
R = R1 + R2.
Это справедливо и для большего количества соединённых последовательно резисторов:
R = R1 + R2 + R3 + R4 + … + Rn.

 

Цепь из последовательно соединённых резисторов будет всегда иметь сопротивление большее, чем у любого резистора из этой цепи.

При последовательном соединении резисторов изменение сопротивления любого резистора из этой цепи влечёт за собой как изменение сопротивления всей цепи так и изменение силы тока в этой цепи.

Параллельное соединение резисторов (формула)

Параллельное соединение резисторов необходимо для уменьшения общего сопротивления и, как вариант, для увеличения мощности нескольких резисторов по сравнению с одним.

Расчет параллельного сопротивления

Расчет параллельного сопротивления двух параллельно соединённых резисторов R1 и R2 производится по следующей формуле:

 

Сопротивление из
двух резисторов:  
R = R1 × R2
 R1 + R2

Параллельное соединение трёх и более резисторов требует более сложной формулы для вычисления общего сопротивления:

Сопротивление параллельных резисторов

 1   =   1 + 1 + 1 + …
RR1R2R3

Как видно, вычислить сопротивление двух параллельных резисторов значительно удобнее.

Сопротивление параллельно соединённых резисторов будет всегда меньше, чем у любого из этих резисторов.

Параллельное соединение резисторов часто используют в случаях, когда необходимо сопротивление с большей мощностью. Для этого, как правило, используют резисторы с одинаковой мощностью и одинаковым сопротивлением. Общая мощность, в таком случае, вычисляется умножением мощности одного резистора на количество параллельно соединённых резисторов.
Например: десять резисторов номиналом 1 КОм и мощностью 1 Вт каждый, соединённые параллельно будут иметь общее сопротивление 100 Ом и мощность 10 Вт.
При последовательном соединении мощность резисторов также складывается. Т.е. в том же примере, но при последовательном соединении, общее сопротивление будет равно 10 КОм и мощность 10 Вт.

Параллельное и последовательное соединение резисторов (сопротивлений)

Все разнообразие схем построено на двух типах соединения — параллельном и последовательном. Для разных соединений действуют разные законы, что и дает возможность создания устройств с различными характеристиками. Рассмотрим последовательное и параллельное соединение резисторов.

Содержание статьи

Что такое резистор и для чего он нужен

Резистор — это радиоэлемент, который увеличивает сопротивление цепи. Ставят его обычно для того, чтобы понизить/ограничить напряжение или ток. Есть сопротивления постоянные и переменные.

Например, светодиоды требуют небольшого тока, иначе перегревается и быстро выходит из строя. Чтобы ограничить ток, перед светодиодом поставьте сопротивление. Ток в цепи станет меньше.

Для чего ставят сопротивления

Для чего нужны резисторы: для подстройки параметров питания

Постоянные сопротивления — это те, которые не меняют своего номинала в процессе работы. Если это и происходит, то считается выходом из строя.

Внешний вид резисторов переменных и постоянных

Так выглядят переменные и постоянные резисторы

Переменные резисторы, наоборот, отличаются тем, что их сопротивление можно изменять. Они имеют бегунок или поворотную ручку, при помощи которых и изменяется номинал. На основе таких устройств делают регуляторы. Например, регулятор громкости, накала греющего элемента и т.д.

Последовательное соединение сопротивлений

Последовательное соединение характеризуется тем, что элементы идут друг за другом. Конец одного подключается к началу другого. При подключении полученной цепочки к источнику тока получается кольцо.

Пример последовательного соединения

Лампы накаливания соединенные последовательно, можно рассматривать как сопротивления

Теоретическая часть

Последовательное соединение характерно тем, что через все элементы протекает ток одинаковой силы. То есть, если цепочка состоит из двух резисторов R1 и R2 (как на рисунке ниже), то ток протекающий через каждое из них и любую другую часть цепи будет одинаковой (I = I1 = I2).

Схема последовательного соединения

Последовательно соединенные сопротивления. I1 — ток протекающий через резистор R1, I2 — ток протекающий через резистор R2

Суммарное сопротивление всей цепи последовательно соединенных резисторов считается как сумма сопротивлений всех ее элементов. То есть, номиналы складывают.

R = R1 + R2 — это и есть формула расчета сопротивления при последовательном соединении резисторов. Если элементов больше двух, будет просто больше слагаемых.

Еще одно свойство последовательного соединения — на каждом элементе напряжение отличается. Ток в цепи одинаковый, а напряжение на резисторе зависит от его номинала.

Примеры расчета

Давайте рассмотрим пример. Цепь представлена на рисунке выше. Есть источник тока и два сопротивления. Пусть R1=1,2 кОм, R2= 800 Ом, а ток в цепи 2 А. По закону Ома U = I * R. Подставляем наши значения:

  • U1 = R1 * I = 1200 Ом * 2 А = 2400 В;
  • U2 = R2 * I = 800 Ом * 2А = 1600 В.

Общее напряжение цепи считается как сумма напряжений на резисторах: U = U1 + U2 = 2400 В + 1600 В = 4000 В.

Иллюстрация последовательного соединения

Так понятнее, что такое последовательное соединение

Полученную цифру можно проверить. Для этого найдем суммарное сопротивление цепи и умножим его на ток.   R = R1 + R2 = 1200 Ом + 800 Ом = 2000 Ом. Если подставить в формулу напряжения при последовательном соединении сопротивлений, получаем: U = R * I = 2000 Ом * 2  А = 4000 В. Получаем, что общее напряжение данной цепи 4000 В.

А теперь посмотрите на схему. На первом вольтметре (возле резистора R1) показания будут 2400 В, на втором  — 1600 В.  При этом напряжение источника питания — 4000 В.

Параллельное соединение резисторов

Параллельное соединение — это когда входы нескольких деталей соединяются в одной точке. Точно так же — в одну точку — соединяют их выходы.

Что такое параллельное соединение

Так выглядит параллельное соединение на схеме и в реальности

Теория и законы параллельного соединения

Если посмотреть на изображение параллельного соединения, заметно, что ко всем элементам прилагается одинаковое напряжение. То есть, при параллельном соединении резисторов, на каждом из них будет одинаковое напряжение.

U = U1 = U2 = U3.

Получается, что ток разделяется на несколько «ручейков». То есть, при параллельном соединении резисторов сила тока, протекающего через каждый из элементов, отличается. I = I1+I2+I3. И зависит сила тока (согласно тому же закону Ома) от сопротивления каждого участка цепи. В случае с параллельным соединением резисторов — от их номинала.

Схема параллельного соединения резисторов

Так выглядит параллельное соединение резисторов на схеме

Общее сопротивление участка цепи при таком соединении становится ниже. Его высчитывают по формуле:

1/R = 1/R1 + 1/R + 1/R3+…

Такая форма хоть и понятна, но неудобна. Формула расчета сопротивления параллельно подключенных резисторов получается тем сложнее, чем больше элементов соединены параллельно. Но больше двух-трех редко кто объединяет, так что на практике достаточно знать только две формулы приведенные ниже.

Формулы расчета: два резистора соединены параллельно и три резистора соединены параллельно

Формулы расчета сопротивления при параллельном подключении двух и трех резисторов

Если подставить значения в эти формулы, то заметим, что результат будет меньше, чем сопротивление резистора с наименьшим номиналом. Это стоит запомнить: результирующее сопротивление включенных параллельно резисторов будет ниже самого маленького номинала.

Примеры расчета параллельного соединения сопротивлений

Давайте сначала рассчитаем параллельное соединение двух резисторов разного номинала и посмотрим что получится.

  • Соединили параллельно 150 Ом и 100 Ом. Считаем результирующее: 150*100 / (150+100) = 15000/250 = 60 Ом.
  • Если соединить 150 Ом и 50 Ом, получим: 150*50 / (150+50) = 7500 / 200 = 37,5 Ом.

Как видим, в обоих случаях результат оказывается меньше чем самый низкий номинал соединенных деталей. Этим и пользуются, если в наличии нет сопротивления небольшого номинала. Проблема только в том, что подбирать сложновато: надо каждый раз считать используя калькулятор.

Формула дял соединения резисторов

Как высчитывать сопротивление составных резисторов

Возможно, вам будет проще, если знать, что соединив два одинаковых резистора параллельно, получим результат в два раза меньше. Например, соединив параллельно два резистора по 100 Ом получим составное сопротивление 50 Ом. Проверим? Считаем: 100*100 / (100+100) = 10000 / 200 = 50 Ом.

Пример параллельного подключения

Еще один пример с лампочками

При соединении параллельно трех резисторов, считать приходится больше, так как формула сложнее. Но картина не отличается:

  • Если подключить параллельно 150 Ом, 100 Ом и 50 Ом, результирующее будет 27,3 Ом.
  • Попробуем с более низкими номиналами. Если параллельно включены 20 Ом, 15 Ом и 10 Ом. Получим результирующее сопротивление 4,61 Ом.

Вот вам подтверждение правила. Суммарное сопротивление параллельно соединенных резисторов меньше чем самый низкий номинал.

Смешанное соединение

Как быть, если в схеме есть и параллельное, и последовательное соединение резисторов? В таком случае считают общее сопротивление по участкам. Можно при этом перерисовывать схему, заменяя составные сопротивления на один «прямоугольник», но проставляя над ним высчитанный результат.

Расчет сопротивления цепи при смешанном соединении резисторов

Пример расчета сопротивления при смешанном соединении резисторов. Рассматриваем исходную схему как совокупность параллельных и последовательных соединений

Шаг 1. Нашли общее сопротивление последовательно соединенных резисторов R3 и R4:

R3-4 = 3 кОм + 3 кОм = 6 кОм;

Шаг 2. Рассчитали сопротивление параллельно соединенных резисторов R2 и R3-4:

R2-4 = 3 кОм * 6 кОм / (3 кОм + 6 кОм) = 18 кОм/9 кОм = 2 кОм;

Шаг 3. Рассчитали общее сопротивление последовательно соединенных резисторов R1 и R2-4:

R1-4 = R1 + R2-4 = 1 кОм + 2 кОм = 3 кОм.

Практическое применение параллельного и последовательного соединения резисторов

Для чего практически можно использовать параллельное и последовательное соединение резисторов? Случается, что при ремонте электронной аппаратуры, не всегда в наличии сопротивление нужного номинала. Ехать в магазин за одним копеечным элементом — накладно. Вот тут и могут пригодиться составные резисторы. Просто надо последовательно или параллельно соединить их, подобрав требуемый номинал.

Применение последовательного и параллельного соединения резисторов

Последовательное и параллельное соединение резисторов применяют для подбора требуемого номинала. Контролировать точное значение получившегося сопротивления можно при помощи цифрового мультиметра

При соединении резисторов, их ножки первоначально скручивают. Какой стороной разворачивать сопротивление — неважно (в отличие от диодов, резисторы одинаково пропускают ток в обоих направлениях). На концах скрутку слегка обжимают плоскогубцами, затем пропаивают. Следите за тем, чтобы корпуса были друг от друга подальше — так они будут лучше охлаждаться при работе.

Общая мощность при последовательном соединении формула

В физике изучается тема про параллельное и последовательное соединение, причем это могут быть не только проводники, но и конденсаторы. Здесь важно не запутаться в том, как выглядит каждое из них на схеме. А уже потом применять конкретные формулы. Их, кстати, нужно помнить наизусть.

Как различить эти два соединения?

Внимательно посмотрите на схему. Если провода представить как дорогу, то машины на ней будут играть роль резисторов. На прямой дороге без каких-либо разветвлений машины едут одна за другой, в цепочку. Так же выглядит и последовательное соединение проводников. Дорога в этом случае может иметь неограниченное количество поворотов, но ни одного перекрестка. Как бы ни виляла дорога (провода), машины (резисторы) всегда будут расположены друг за другом, по одной цепочке.

Совсем другое дело, если рассматривается параллельное соединение. Тогда резисторы можно сравнить со спортсменами на старте. Они стоят каждый на своей дорожке, но направление движения у них одинаковое, и финиш в одном месте. Так же и резисторы — у каждого из них свой провод, но все они соединены в некоторой точке.

Формулы для силы тока

О ней всегда идет речь в теме «Электричество». Параллельное и последовательное соединение по-разному влияют на величину силы тока в резисторах. Для них выведены формулы, которые можно запомнить. Но достаточно просто запомнить смысл, который в них вкладывается.

Так, ток при последовательном соединении проводников всегда одинаков. То есть в каждом из них значение силы тока не отличается. Провести аналогию можно, если сравнить провод с трубой. В ней вода течет всегда одинаково. И все препятствия на ее пути будут сметаться с одной и той же силой. Так же с силой тока. Поэтому формула общей силы тока в цепи с последовательным соединением резисторов выглядит так:

Здесь буквой I обозначена сила тока. Это общепринятое обозначение, поэтому его нужно запомнить.

Ток при параллельном соединении уже не будет постоянной величиной. При той же аналогии с трубой получается, что вода разделится на два потока, если у основной трубы будет ответвление. То же явление наблюдается с током, когда на его пути появляется разветвление проводов. Формула общей силы тока при параллельном соединении проводников:

Если разветвление составлено из проводов, которых больше двух, то в приведенной формуле на такое же количество станет больше слагаемых.

Формулы для напряжения

Когда рассматривается схема, в которой выполнено соединение проводников последовательно, то напряжение на всем участке определяется суммой этих величин на каждом конкретном резисторе. Сравнить эту ситуацию можно с тарелками. Удержать одну из них легко получится одному человеку, вторую рядом он тоже сможет взять, но уже с трудом. Держать в руках три тарелки рядом друг с другом одному человеку уже не удастся, потребуется помощь второго. И так далее. Усилия людей складываются.

Формула для общего напряжения участка цепи с последовательным соединением проводников выглядит так:

Другая ситуация складывается, если рассматривается параллельное соединение резисторов. Когда тарелки ставятся друг на друга, их по-прежнему может удержать один человек. Поэтому складывать ничего не приходится. Такая же аналогия наблюдается при параллельном соединении проводников. Напряжение на каждом из них одинаковое и равно тому, которое на всех них сразу. Формула общего напряжения такая:

Формулы для электрического сопротивления

Их уже можно не запоминать, а знать формулу закона Ома и из нее выводить нужную. Из указанного закона следует, что напряжение равно произведению силы тока и сопротивления. То есть U = I * R, где R — сопротивление.

Тогда формула, с которой нужно будет работать, зависит от того, как выполнено соединение проводников:

  • последовательно, значит, нужно равенство для напряжения — Iобщ * Rобщ = I1 * R1 + I2 * R2;
  • параллельно необходимо пользоваться формулой для силы тока — Uобщ / Rобщ = U1 / R1 + U2 / R2 .

Далее следуют простые преобразования, которые основываются на том, что в первом равенстве все силы тока имеют одинаковое значение, а во втором — напряжения равны. Значит, их можно сократить. То есть получаются такие выражения:

  1. R общ = R 1 + R 2 (для последовательного соединения проводников).
  2. 1 / R общ = 1 / R 1 + 1 / R 2 (при параллельном соединении).

При увеличении числа резисторов, которые включены в сеть, изменяется количество слагаемых в этих выражениях.

Стоит отметить, что параллельное и последовательное соединение проводников по-разному влияют на общее сопротивление. Первое из них уменьшает сопротивление участка цепи. Причем оно оказывается меньше самого маленького из использованных резисторов. При последовательном соединении все логично: значения складываются, поэтому общее число всегда будет самым большим.

Работа тока

Предыдущие три величины составляют законы параллельного соединения и последовательного расположения проводников в цепи. Поэтому их знать нужно обязательно. Про работу и мощность необходимо просто запомнить базовую формулу. Она записывается так: А = I * U * t, где А — работа тока, t — время его прохождения по проводнику.

Для того чтобы определить общую работу при последовательном соединении нужно заменить в исходном выражении напряжение. Получится равенство: А = I * (U 1 + U 2) * t, раскрыв скобки в котором получится, что работа на всем участке равна их сумме на каждом конкретном потребителе тока.

Аналогично идет рассуждение, если рассматривается схема параллельного соединения. Только заменять полагается силу тока. Но результат будет тот же: А = А 1 + А 2.

Мощность тока

При выведении формулы для мощности (обозначение «Р») участка цепи опять нужно пользоваться одной формулой: Р = U * I. После подобных рассуждений получается, что параллельное и последовательное соединение описываются такой формулой для мощности: Р = Р 1 + Р 2.

То есть, как бы ни были составлены схемы, общая мощность будет складываться из тех, которые задействованы в работе. Именно этим объясняется тот факт, что нельзя включать в сеть квартиры одновременно много мощных приборов. Она просто не выдержит такой нагрузки.

Как влияет соединение проводников на ремонт новогодней гирлянды?

Сразу же после того, как перегорит одна из лампочек, станет ясно, как они были соединены. При последовательном соединении не будет светиться ни одна из них. Это объясняется тем, что пришедшая в негодность лампа создает разрыв в цепи. Поэтому нужно проверить все, чтобы определить, какая перегорела, заменить ее – и гирлянда станет работать.

Если в ней используется параллельное соединение, то она не перестает работать при неисправности одной из лампочек. Ведь цепь не будет полностью разорвана, а только одна параллельная часть. Чтобы отремонтировать такую гирлянду, не нужно проверять все элементы цепи, а только те, которые не светятся.

Что происходит с цепью, если в нее включены не резисторы, а конденсаторы?

При их последовательном соединении наблюдается такая ситуация: заряды от плюсов источника питания поступают только на внешние обкладки крайних конденсаторов. Те, что находятся между ними, просто передают этот заряд по цепочке. Этим объясняется то, что на всех обкладках появляются одинаковые заряды, но имеющие разные знаки. Поэтому электрический заряд каждого конденсатора, соединенного последовательно, можно записать такой формулой:

Для того чтобы определить напряжение на каждом конденсаторе, потребуется знание формулы: U = q / С. В ней С — емкость конденсатора.

Общее напряжение подчиняется тому же закону, который справедлив для резисторов. Поэтому, заменив в формуле емкости напряжение на сумму, мы получим, что общую емкость приборов нужно вычислять по формуле:

Упростить эту формулу можно, перевернув дроби и заменив отношение напряжения к заряду емкостью. Получается такое равенство: 1 / С = 1 / С 1 + 1 / С 2.

Несколько по-другому выглядит ситуация, когда соединение конденсаторов — параллельное. Тогда общий заряд определяется суммой всех зарядов, которые накапливаются на обкладках всех приборов. А значение напряжения по-прежнему определяется по общим законам. Поэтому формула для общей емкости параллельно соединенных конденсаторов выглядит так:

С = (q 1 + q 2 ) / U.

То есть эта величина считается, как сумма каждого из использованных в соединении приборов:

Как определить общее сопротивление произвольного соединения проводников?

То есть такого, в котором последовательные участки сменяют параллельные, и наоборот. Для них по-прежнему справедливы все описанные законы. Только применять их нужно поэтапно.

Сперва полагается мысленно развернуть схему. Если представить ее сложно, то нужно нарисовать то, что получается. Объяснение станет понятнее, если рассмотреть его на конкретном примере (см. рисунок).

Ее удобно начать рисовать с точек Б и В. Их необходимо поставить на некотором удалении друг от друга и от краев листа. Слева к точке Б подходит один провод, а вправо направлены уже два. Точка В, напротив, слева имеет два ответвления, а после нее расположен один провод.

Теперь необходимо заполнить пространство между этими точками. По верхнему проводу нужно расположить три резистора с коэффициентами 2, 3 и 4, а снизу пойдет тот, у которого индекс равен 5. Первые три соединены последовательно. С пятым резистором они параллельны.

Оставшиеся два резистора (первый и шестой) включены последовательно с рассмотренным участком БВ. Поэтому рисунок можно просто дополнить двумя прямоугольниками по обе стороны от выбранных точек. Осталось применить формулы для расчета сопротивления:

  • сначала ту, которая приведена для последовательного соединения;
  • потом для параллельного;
  • и снова для последовательного.

Подобным образом можно развернуть любую, даже очень сложную схему.

Задача на последовательное соединение проводников

Условие. В цепи друг за другом подсоединены две лампы и резистор. Общее напряжение равно 110 В, а сила тока 12 А. Чему равно сопротивление резистора, если каждая лампа рассчитана на напряжение в 40 В?

Решение. Поскольку рассматривается последовательное соединение, формулы его законов известны. Нужно только правильно их применить. Начать с того, чтобы выяснить значение напряжения, которое приходится на резистор. Для этого из общего нужно вычесть два раза напряжение одной лампы. Получается 30 В.

Теперь, когда известны две величины, U и I (вторая из них дана в условии, так как общий ток равен току в каждом последовательном потребителе), можно сосчитать сопротивление резистора по закону Ома. Оно оказывается равным 2,5 Ом.

Ответ. Сопротивление резистора равно 2,5 Ом.

Задача на соединение конденсаторов, параллельное и последовательное

Условие. Имеются три конденсатора с емкостями 20, 25 и 30 мкФ. Определите их общую емкость при последовательном и параллельном соединении.

Решение. Проще начать с параллельного подключения. В этой ситуации все три значения нужно просто сложить. Таким образом, общая емкость оказывается равной 75 мкФ.

Несколько сложнее расчеты будут при последовательном соединении этих конденсаторов. Ведь сначала нужно найти отношения единицы к каждой из этих емкостей, а потом сложить их друг с другом. Получается, что единица, деленная на общую емкость, равна 37/300. Тогда искомая величина получается приблизительно 8 мкФ.

Ответ. Общая емкость при последовательном соединении 8 мкФ, при параллельном — 75 мкФ.

Проводники в электрических цепях могут соединяться последовательно и параллельно.

При последовательном соединении проводников (рис. 1.9.1) сила тока во всех проводниках одинакова:

Рисунок 1.9.1.

По закону Ома, напряжения и на проводниках равны

Общее напряжение на обоих проводниках равно сумме напряжений 1 и 2:

где – электрическое сопротивление всей цепи. Отсюда следует:

При последовательном соединении полное сопротивление цепи равно сумме сопротивлений отдельных проводников.

Этот результат справедлив для любого числа последовательно соединенных проводников.

При параллельном соединении (рис. 1.9.2) напряжения 1 и 2 на обоих проводниках одинаковы:

Сумма токов 1 + 2, протекающих по обоим проводникам, равна току в неразветвленной цепи:

Этот результат следует из того, что в точках разветвления токов (узлы и ) в цепи постоянного тока не могут накапливаться заряды. Например, к узлу за время Δ подтекает заряд Δ, а утекает от узла за то же время заряд 1Δ + 2Δ. Следовательно, = 1 + 2.

Рисунок 1.9.2.

Записывая на основании закона Ома

где – электрическое сопротивление всей цепи, получим

При параллельном соединении проводников величина, обратная общему сопротивлению цепи, равна сумме величин, обратных сопротивлениям параллельно включенных проводников.

Этот результат справедлив для любого числа параллельно включенных проводников.

Формулы для последовательного и параллельного соединения проводников позволяют во многих случаях рассчитывать сопротивление сложной цепи, состоящей из многих резисторов. На рис. 1.9.3 приведен пример такой сложной цепи и указана последовательность вычислений.

Рисунок 1.9.3.

Следует отметить, что далеко не все сложные цепи, состоящие из проводников с различными сопротивлениями, могут быть рассчитаны с помощью формул для последовательного и параллельного соединения. На рис. 1.9.4 приведен пример электрической цепи, которую нельзя рассчитать указанным выше методом.

Рисунок 1.9.4.

Цепи, подобные изображенной на рис. 1.9.4, а также цепи с разветвлениями, содержащие несколько источников, рассчитываются с помощью правил Кирхгофа.

Все известные виды проводников обладают определенными свойствами, в том числе и электрическим сопротивлением. Это качество нашло свое применение в резисторах, представляющих собой элементы цепи с точно установленным сопротивлением. Они позволяют выполнять регулировку тока и напряжения с высокой точностью в схемах. Все подобные сопротивления имеют свои индивидуальные качества. Например, мощность при паралл ельном и последовательном соединении резисторов будет различной. Поэтому на практике очень часто используются различные методики расчетов, благодаря которым возможно получение точных результатов.

Свойства и технические характеристики резисторов

Как уже отмечалось, резисторы в электрических цепях и схемах выполняют регулировочную функцию. С этой целью используется закон Ома, выраженный формулой: I = U/R. Таким образом, с уменьшением сопротивления происходит заметное возрастание тока. И, наоборот, чем выше сопротивление, тем меньше ток. Благодаря этому свойству, резисторы нашли широкое применение в электротехнике. На этой основе создаются делители тока, использующиеся в конструкциях электротехнических устройств.

Помимо функции регулировки тока, резисторы применяются в схемах делителей напряжения. В этом случае закон Ома будет выглядеть несколько иначе: U = I x R. Это означает, что с ростом сопротивления происходит увеличение напряжения. На этом принципе строится вся работа устройств, предназначенных для деления напряжения. Для делителей тока используется паралл ельное соединение резисторов, а для делителей напряжения – последовательное.

На схемах резисторы отображаются в виде прямоугольника, размером 10х4 мм. Для обозначения применяется символ R, который может быть дополнен значением мощности данного элемента. При мощности свыше 2 Вт, обозначение выполняется с помощью римских цифр. Соответствующая надпись наносится на схеме возле значка резистора. Мощность также входит в состав маркировки, нанесенной на корпус элемента. Единицами измерения сопротивления служат ом (1 Ом), килоом (1000 Ом) и мегаом (1000000 Ом). Ассортимент резисторов находится в пределах от долей ома до нескольких сотен мегаом. Современные технологии позволяют изготавливать данные элементы с довольно точными значениями сопротивления.

Важным параметром резистора считается отклонение сопротивления. Его измерение осуществляется в процентах от номинала. Стандартный ряд отклонений представляет собой значения в виде: +20, +10, +5, +2, +1% и так далее до величины +0,001%.

Большое значение имеет мощность резистора. По каждому из них во время работы проходит электрический ток, вызывающий нагрев. Если допустимое значение рассеиваемой мощности превысит норму, это приведет к выходу из строя резистора. Следует учитывать, что в процессе нагревания происходит изменение сопротивления элемента. Поэтому если устройства работают в широких диапазонах температур, применяется специальная величина, именуемая температурным коэффициентом сопротивления.

Для соединения резисторов в схемах используются три разных способа подключения – паралл ельное, последовательное и смешанное. Каждый способ обладает индивидуальными качествами, что позволяет применять данные элементы в самых разных целях.

Мощность при последовательном соединение

При соединение резисторов последовательно электрический ток по очереди проходит через каждое сопротивление. Значение тока в любой точке цепи будет одинаковым. Данный факт определяется с помощью закона Ома. Если сложить все сопротивления, приведенные на схеме, то получится следующий результат: R = 200+100+51+39 = 390 Ом.

Учитывая напряжение в цепи, равное 100 В, по закону Ома сила тока будет составлять I = U/R = 100/390 = 0,256 A. На основании полученных данных можно рассчитать мощность резисторов при последовательном соединении по следующей формуле: P = I 2 x R = 0,256 2 x 390 = 25,55 Вт.

Таким же образом можно рассчитать мощность каждого отдельно взятого резистора:

  • P1 = I 2 x R1 = 0,256 2 x 200 = 13,11 Вт;
  • P2 = I 2 x R2 = 0,256 2 x 100 = 6,55 Вт;
  • P3 = I 2 x R3 = 0,256 2 x 51 = 3,34 Вт;
  • P4 = I 2 x R4 = 0,256 2 x 39 = 2,55 Вт.

Если сложить полученные мощность, то полная Р составит: Р = 13,11+6,55+3,34+2,55 = 25,55 Вт.

Мощность при паралл ельном соединение

При паралл ельном подключении все начала резисторов соединяются с одним узлом схемы, а концы – с другим. В этом случае происходит разветвление тока, и он начинает протекать по каждому элементу. В соответствии с законом Ома, сила тока будет обратно пропорциональна всем подключенным сопротивлениям, а значение напряжения на всех резисторах будет одним и тем же.

Прежде чем вычислять силу тока, необходимо выполнить расчет полной проводимости всех резисторов, применяя следующую формулу:

  • 1/R = 1/R1+1/R2+1/R3+1/R4 = 1/200+1/100+1/51+1/39 = 0,005+0,01+0,0196+0,0256 = 0,06024 1/Ом.
  • Поскольку сопротивление является величиной, обратно пропорциональной проводимости, его значение составит: R = 1/0,06024 = 16,6 Ом.
  • Используя значение напряжения в 100 В, по закону Ома рассчитывается сила тока: I = U/R = 100 x 0,06024 = 6,024 A.
  • Зная силу тока, мощность резисторов, соединенных паралл ельно, определяется следующим образом: P = I 2 x R = 6,024 2 x 16,6 = 602,3 Вт.
  • Расчет силы тока для каждого резистора выполняется по формулам: I1 = U/R1 = 100/200 = 0,5A; I2 = U/R2 = 100/100 = 1A; I3 = U/R3 = 100/51 = 1,96A; I4 = U/R4 = 100/39 = 2,56A. На примере этих сопротивлений прослеживается закономерность, что с уменьшением сопротивления, сила тока увеличивается.

Существует еще одна формула, позволяющая рассчитать мощность при паралл ельном подключении резисторов: P1 = U 2 /R1 = 100 2 /200 = 50 Вт; P2 = U 2 /R2 = 100 2 /100 = 100 Вт; P3 = U 2 /R3 = 100 2 /51 = 195,9 Вт; P4 = U 2 /R4 = 100 2 /39 = 256,4 Вт. Сложив мощности отдельных резисторов, получится их общая мощность: Р = Р1234 = 50+100+195,9+256,4 = 602,3 Вт.

Таким образом, мощность при последовательном и паралл ельном соединении резисторов определяется разными способами, с помощью которых можно получить максимально точные результаты.

Резисторы в последовательном и параллельном соединении

Резисторы, включенные последовательно и параллельно

В нашем предыдущем посте о резисторах мы изучили различные типы резисторов.

В некоторых случаях, когда мы не получаем желаемые или конкретные значения резисторов, мы должны либо использовать переменные резисторы, такие как потенциометры или предварительные настройки, чтобы получить такие точные значения. Однако такие горшки слишком дороги в использовании на все случаи жизни.

Другой способ сделать это — объединить два или более резистора для получения необходимых точных значений.Такие комбинации резисторов стоят очень дешевле.

Теперь возникает вопрос, как комбинировать эти резисторы.

Резисторы можно комбинировать двумя способами, например:

    Комбинации серии
  1. Параллельные комбинации
Резисторы

серии

Считается, что резисторы

подключены в « Series », когда они последовательно соединены в одну линию.

Расчет значений для двух или более резисторов, соединенных последовательно, просто, просто сложите все значения.

Последовательное соединение гарантирует, что ТО ЖЕ ток течет через все резисторы.

В этом типе подключения R Итого всегда будет БОЛЬШЕ, чем любой из включенных резисторов.

Общее сопротивление — это сумма всех резисторов, включенных последовательно, и определяется выражением:

рупий Итого = 1 + 2 + 3 + …………

Пример:

Resistors in series

  • Поскольку резисторы соединены последовательно, один и тот же ток проходит через каждый резистор в цепи, и общее сопротивление R Всего цепи должно быть равным к сумме всех отдельных резисторов, сложенных вместе.Это

рупий Итого = 1 + 2

  • Общее приложенное напряжение V делится на два резистора.
  • Ток в цепи определяется как:

2

  • Используя закон Ома, напряжения на R 1 и R 2 задаются как:

3

4

  • Следовательно, полное напряжение определяется как:

5

  • Например, если мы возьмем V = 6 В, R 1 = 1 кОм и R 2 = 2 кОм, тогда

R Всего = 1 кОм + 2 кОм = 3 кОм

I = 6 В / 3 кОм = 2 мА

Напряжение на резисторе 1 кОм составляет В 1 = 2 мА × 1 кОм = 2 В

Напряжение на резисторе 2 кОм составляет В 2 = 2 мА × 2 кОм = 4 В

Итак, мы видим, что мы можем заменить два отдельных резистора, указанных выше, только одним единственным «эквивалентным» резистором, который будет иметь номинал 3 кОм.

Это полное сопротивление обычно известно как эквивалентное сопротивление и может быть определено как; « — единое значение сопротивления, которое может заменить любое количество последовательно подключенных резисторов без изменения значений тока или напряжения в цепи ».

Последовательное соединение можно охарактеризовать следующими пунктами:

  1. Одинаковый ток протекает через все последовательно включенные резисторы.
  2. Результирующий резистор представляет собой СУММУ всех последовательно включенных резисторов.
  3. Резисторы серии
  4. делят общее приложенное напряжение пропорционально их величине.

Схема делителя напряжения

Поскольку последовательные резисторы делят напряжение, эту идею можно использовать для получения меньшего напряжения с выхода источника питания.

Например, у нас есть блок питания с фиксированным выходом 10 В. Но мы хотим от него только 5В. Как его получить ?

voltage divider circuit

Схема, показанная выше, состоит из двух резисторов, R 1 и R 2 , соединенных последовательно через напряжение питания V в .

Ток I определяется по формуле:

11

Поскольку ток I протекает через R 1 , а также через R 2 , следовательно, используя закон Ома, напряжение, развиваемое на R 2 , определяется как:

12

13

Если R 1 = R 2 , то V out = V дюйм /2

Если к цепи последовательно подключено больше резисторов, то на каждом резисторе по очереди будут появляться разные напряжения в зависимости от их индивидуальных значений сопротивления, обеспечивая разные, но меньшие точки напряжения от одного источника.

Параллельные резисторы

Считается, что резисторы

соединены вместе в « параллельном, », когда оба их вывода соответственно подключены к каждому выводу другого резистора или резисторов.

Параллельная комбинация

Рис. ниже показана схема параллельной комбинации резисторов, где два резистора R 1 и R 2 подключены параллельно через напряжение питания E.

resistors in parallel

Как видно из рис.выше:

  • Для Current доступны два пути. Следовательно, ток разделяется.
  • А вот напряжения на резисторах такие же.
  • Если два резистора равны, ток будет делиться поровну, и R Total будет ровно половиной любого резистора или ровно одной третью, если есть три равных резистора.
  • В целом можно сказать:

10

7

Токи в параллельной цепи резистора

В цепи параллельных резисторов напряжение на каждом резисторе, подключенном параллельно, остается неизменным.Однако ток через каждый параллельный резистор не обязательно будет одинаковым, поскольку значение сопротивления в каждой ветви определяет ток в этой ветви.

Общий ток I Total в параллельной цепи резистора — это сумма отдельных токов, протекающих во всех параллельных ветвях, которая может быть определена с помощью закона Ома.

Пример

resistor in parallel example

Допустим, напряжение E равно 6 В.

Резисторы должны быть R 1 = 1 кОм и R 2 = 2 кОм.

Согласно закону Ома, ток через R 1 = 6 В / 1 кОм = 6 мА и ток через R 2 = 6 В / 2 кОм = 3 мА

Следовательно, общий ток составляет 6 мА + 3 мА = 9 мА

6 В будет генерировать 9 мА только тогда, когда полное сопротивление цепи равно:

6 В / 9 мА = 0,66 кОм

Следовательно, эффективное сопротивление параллельно соединенных R 1 и R 2 составляет 0,66 кОм.

Это эффективное сопротивление также можно рассчитать по следующей формуле:

8

9

Таким образом, параллельное соединение можно охарактеризовать:

  1. На всех параллельно подключенных резисторах присутствует одинаковое напряжение.
  2. Обратное значение результирующего или полного сопротивления является суммой обратных величин всех резисторов, включенных параллельно.
  3. Параллельные резисторы делят общий ток обратно пропорционально их величине.
  4. Когда набор резисторов соединен параллельно, эффективное сопротивление всегда меньше, чем наименьшее в наборе.

Например: пусть резисторы 1 кОм и 10 кОм включены параллельно.

Тогда результат будет (1 k × 10 k) / 11 k = 0.9 кОм, что меньше 1 кОм (наименьшее).

Последовательные и параллельные резисторы

В некоторых электрических и электронных схемах требуется соединять различные резисторы вместе в «ОБЕИХ» параллельных и последовательных комбинациях в одной и той же цепи и создавать более сложные резистивные цепи.

Теперь возникает вопрос, как рассчитать комбинированное или полное сопротивление цепи, токи и напряжения для этих резистивных комбинаций.

Цепи резисторов

, которые объединяют последовательно и параллельные цепи резисторов вместе, обычно известны как комбинации резисторов или схемы со смешанными резисторами.

Метод расчета эквивалентного сопротивления цепей такой же, как и для любой отдельной последовательной или параллельной цепи.

Самая важная вещь, которую следует иметь в виду при таких расчетах, — это то, что последовательно подключенные резисторы несут точно такой же ток и что резисторы, подключенные параллельно, имеют точно такое же напряжение на них.

Пример

Рассмотрим схему, изображенную на рис. ниже:

resoistor combinations

В приведенной выше схеме рассчитаем полный ток (I T ), снимаемый с источника питания 12 В.

Мы видим, что два резистора, R 2 и R 3 фактически соединены в комбинации «СЕРИЯ», поэтому мы можем сложить их вместе, чтобы получить эквивалентное сопротивление. Таким образом, результирующее сопротивление для этой комбинации будет:

R 2 + R 3 = 8 Ом +4 Ом = 12 Ом

Таким образом, мы можем заменить оба резистора R 2 и R 3 , указанные выше, на один резистор с сопротивлением 12 Ом, как показано на рис. ниже:

15

Итак, наша схема теперь имеет единственный резистор R A в «ПАРАЛЛЕЛЬНО» с резистором R 4 .Используя наши резисторы в параллельном уравнении, мы можем уменьшить эту параллельную комбинацию до одного эквивалентного резистора номиналом R (комбинация) , используя следующую формулу для двух параллельно соединенных резисторов.

16

17

Резистивная цепь теперь выглядит примерно так:

18

Мы можем видеть, что два оставшихся сопротивления, R 1 и R (комбинация) соединены вместе в комбинации «ПОСЛЕДОВАТЕЛЬНОСТЬ», и снова их можно сложить (резисторы, включенные последовательно), так что общее сопротивление цепи приведено как:

19

Одно сопротивление всего 12 Ом можно использовать для замены четырех исходных резисторов, соединенных вместе в исходной цепи.

20

Теперь, используя закон Ома, значение тока цепи (I) просто рассчитывается как:

21

,

Комбинация резисторов — последовательно и параллельно — Учебный материал для IIT JEE

  • Полный курс физики — 11 класс
  • ПРЕДЛАГАЕМАЯ ЦЕНА: рупий.2968

  • Просмотр подробностей

 


Введение в сочетание резисторов — последовательных и параллельных

Электрический ток — это поток заряженных частиц.Поток зарядов будет постоянным в текущей электроэнергии. Электрический ток течет от более высокого электрического потенциала к более низкому электрическому потенциалу. Для протекания тока требуется замкнутый контур из проводящего материала. Схема состоит из проводов, соединенных встык, и электроны движутся в одном направлении.

Схема имеет жилы (провод), выключатель, нагрузку и источник питания. Схема начинается и останавливается в одной и той же точке. Обычно медные провода используются как жилы без изоляции.Именно через проводник течет ток. Переключатель используется для размыкания или замыкания цепи. Когда переключатель замкнут, ток течет по цепи, а когда переключатель разомкнут, он размыкает цепь, и ток через нее не течет. Клетка может быть источником энергии. Если мы поместим более одной ячейки, она станет батареей.

Нагрузка, также известная как Резистор , использует электрическую энергию и преобразует ее в другую форму энергии. Это может быть лампочка или что-нибудь еще.Если в цепи нет нагрузки, произойдет короткое замыкание.

A circuit

Схема А

Параметры цепи

Электрический ток измеряется в амперах (амперах) амперметром, который подключается последовательно с другими компонентами в цепи. Ток, I = Q / t, где Q — заряд в кулонах, а t — время в секундах. Андре Мари Ампер обнаружил, что два параллельных провода притягиваются друг к другу, когда электрический ток течет в одном направлении.Кроме того, два параллельных провода отталкиваются друг от друга, когда электрический ток течет в противоположных направлениях. В результате его открытий в этой области, единица измерения тока была получена от его имени, которое называется « amp ». Один ампер равен одному кулону заряда в секунду времени.

Напряжение определяется как разность электрических потенциалов между двумя точками в цепи. Единица измерения напряжения — вольт. Устройство происходит от имени Алессандро Вольта. Элементы или батареи обеспечивают необходимое напряжение или разность потенциалов.

Сопротивление препятствует прохождению тока. Это мера способности объекта удерживать поток электронов. Сопротивление будет низким в проводнике и высоким в изоляторе. Измеряется в омах. Единица ома названа в честь ученого Георга Симона Ома, сформулировавшего закон Ома.

Резисторы используются для управления прохождением электрического тока в цепи. Он преобразует электрическую энергию в тепло и свет. Резистор является пассивным компонентом, поскольку он потребляет мощность, но не генерирует ее.Обычно они состоят из металла, углерода или пленки оксида металла. Резисторы используются для ограничения тока и защиты полупроводниковых устройств, таких как светодиоды. Он также используется для ограничения частотной характеристики в цепи фильтра.

Закон Ома

Георг Симон Ом показал взаимосвязь между напряжением, током и сопротивлением и сформулировал закон Ома. Этот закон — основа электричества.

Закон гласит, что V = I R, где напряжение V выражено в вольтах, ток I — в амперах, а сопротивление R — в омах.

Таким образом, I = V / R и R = V / I.

Для большей ясности электричество можно отнести к воде. Таким образом, напряжение в цепи, единицей измерения которой являются вольты, идентично давлению воды, текущей в трубе. Ток в цепи, где единица измерения равен амперам, эквивалентен току воды, протекающей по трубе. Сопротивление в контуре, измеряемое в омах, совпадает с сопротивлением трения и размером трубы, которая ограничивает поток воды.

Ohm’s Law

Закон Ома

Серия и параллельное соединение

Существует два основных типа цепей: последовательные и параллельные.И последовательная, и параллельная цепи состоят из более чем одной нагрузки. Резисторы могут быть подключены последовательно, параллельно или их комбинация.

В последовательной цепи электронов движутся только по одному пути. Здесь будет тот же ток, который проходит через каждый резистор. Напряжение на резисторе при последовательном включении будет другим. При последовательном соединении, если один резистор сломан или возникает какая-либо неисправность, вся цепь выключается. Последовательные цепи нелегко перегреть.Конструкция последовательной схемы проста по сравнению с параллельной схемой.

Некоторые огни рождественской елки могут быть включены в последовательные цепи. Если погаснет одна лампочка, погаснет вся струна. Предохранитель или автоматические выключатели будут подключены последовательно, чтобы защитить всю проводку от перегрузки по току. Его можно использовать как делитель напряжения. Батарейки в пульте подключены последовательно.

Series circuit

Последовательная цепь

В параллельной цепи электронов проходят через множество ее ветвей.В этом случае напряжение на всех резисторах в цепи остается неизменным. Здесь ток в цепи делится между каждой ветвью и, наконец, рекомбинирует, когда ветви встречаются в общей точке. Параллельная цепь может быть сформирована разными способами, что означает, что резисторы могут быть расположены в разных формах. Его можно использовать как делитель тока.

В большинстве случаев цепи подключаются параллельно. Это потому, что если один резистор сломан или поврежден, он не отключит всю систему.Но из-за этого эффекта трудно обнаружить отказ, если в цепи что-то пойдет не так, и поэтому в определенные моменты это может быть опасно. Легко подключить или отключить новый резистор или другой компонент, не влияя на другие элементы в параллельной цепи. Но он использует много проводов и, следовательно, становится сложным. В основном в зданиях и домах мы используем параллельное подключение.

Parallel circuit

Параллельная цепь

Комбинация резисторов при последовательном включении

Рассмотрим три резистора R 1 , R 2 , R 3 , которые включены последовательно.Здесь заряд сначала проходит через R 1 , затем входит в R 2 и, наконец, достигает R 3 .

Combination of three resistors in series

Комбинация из трех последовательно соединенных резисторов

По закону Ома разность потенциалов на R 1 = V 1 = I R 1

Разность потенциалов на R 2 = V 2 = I R 2 .

Разность потенциалов на R 3 = V 3 = I R 3 .

Таким образом, разность потенциалов V на этом последовательном соединении резисторов

В = В 1 + В 2 + В 3

= I R 1 + I R 2 + I R 3

= I ( 1 + 2 + 3 )

Таким образом, в случае последовательного соединения эквивалентное сопротивление R eq = V / I = (R 1 + R 2 + R 3 ).

Для n числа резисторов, включенных последовательно, эквивалентное сопротивление R eq = R 1 + R 2 + R 3 ………………… R n .

Эквивалентное сопротивление — это полное сопротивление цепи. Это единственное значение сопротивления, которое может заменить количество резисторов в цепи без изменения тока и напряжения в сети. Таким образом, при последовательном соединении общее сопротивление цепи определяется путем сложения сопротивлений каждого отдельного резистора.

Для примера рассмотрим последовательную схему, состоящую из трех резисторов с сопротивлением 5 Ом, 10 Ом, 5 Ом соответственно с батареей 15 В.

Итак, полное сопротивление R = R 1 + R 2 + R 3 = 5 + 10 + 5 = 20 Ом

Мы знаем, что V = I R.

Ток I = V / R = 15/20 = 0,75 А. Хотя ток в последовательной сети одинаков, падение напряжения на каждом резисторе разное. Каждый резистор с разным сопротивлением обеспечивает разное падение напряжения, и мы можем найти полное напряжение по закону Ома, V = I R. Давайте возьмем предыдущий пример. Упомянутое здесь напряжение составляет 15 В.Мы можем проверить это, вычислив таким образом. V = I R 1 + I R 2 + I R 3 = 0,75 (5 +10 + 5) = 15 В. Это точное измерение напряжения, которое мы предоставили в этом примере. Обнаружено, что резистор с большим сопротивлением имеет большее падение напряжения.


Комбинация резисторов при параллельном включении

Рассмотрим три резистора R 1 , R 2 , R 3 , включенных параллельно. Заряд делится на три и проходит через 1 рандов, 2 рандов и рандов 3.

Combination of three resistors in parallel

Комбинация из трех параллельно включенных резисторов

Ток I = I 1 + I 2 + I 3.

Разность потенциалов, приложенная к R1 = V = I 1 R 1

Разность потенциалов на R 2 = V = I 2 R 2

Разность потенциалов на R 3 = V = I 3 R 3

Таким образом, I = I 1 + I 2 + I 3

= V / R 1 + V / R 2 + V / R 3

= V (1 / R 1 + 1 / R 2 + 1 / R 3 )

Если эту параллельную комбинацию заменить эквивалентным сопротивлением, R eq

Тогда I = V / R экв

1 / R экв = 1 / R 1 + 1 / R 2 + 1 / R 3

Таким образом, для n количества резисторов, включенных параллельно, 1 / R eq = 1 / R 1 + 1 / R 2 + 1 / R 3 ………………………. 1 / R .

Таким образом, при параллельном соединении общее сопротивление цепи определяется путем сложения обратной величины сопротивления каждого отдельного резистора.

Для примера рассмотрим параллельную схему, состоящую из трех резисторов с сопротивлением 5 Ом, 10 Ом, 5 Ом соответственно с батареей 15 В.

Таким образом, общее сопротивление, 1 / R = 1 / R 1 + 1 / R 2 + 1 / R 3 = 1/5 + 1/10 + 1/5 = 5/10. Итак, R = 2Ω.

При параллельном подключении общее сопротивление или эквивалентное сопротивление всегда будет меньше наименьшего резистора, присутствующего в цепи. Значение эквивалентного сопротивления будет между наименьшим сопротивлением в цепи и наименьшим сопротивлением, деленным на количество резисторов, присутствующих в цепи. В этом примере наименьший резистор имеет сопротивление 5 Ом, а значение общего сопротивления составляет 2 Ом, что явно подтверждает вышеупомянутый факт.

Напряжение на каждом резисторе составляет 15 В.Теперь, чтобы найти ток через каждую ветвь, формула I = V / R.

I 1 = 15/5 = 3A

I 2 = 15/10 = 1,5 A

I 3 = 15/5 = 3A

Общий ток I = 3 + 1,5 + 3 = 7,5 A


Комбинация последовательных и параллельных резисторов

Рассмотрим схему, в которой R 2 и R 3 подключены параллельно, а R 1 включены последовательно с R 2 и R 3.

Combination of series and parallel resistors

Комбинация последовательных и параллельных резисторов


Сначала рассмотрим R 2 и R 3 и, таким образом, 1 / R 23eq = 1 / R 2 + 1 / R 3

R 23eq = R 2 R 3 / R 2 + R 3

R 123eq = R 23eq + R 1

Таким образом, ток I = V / R 123eq = V / [R 1 + (R 2 R 3 / R 2 + R 3 )]

= V (R 2 + R 3 ) / R 1 R 2 + R 1 R 3 + R 2 R 3.

Сводка

  • Электрический ток — это поток заряженных частиц в цепи. Схема состоит из проводника, резистора, переключателя и источника питания.

  • Ток, I = Q / t, где Q — заряд в кулонах, а t — время в секундах. Напряжение — это разность электрических потенциалов между двумя точками в цепи. Вольт — это единица измерения напряжения.

  • Сопротивление — это способность управлять потоком электронов в цепи, а единицей измерения сопротивления являются омы.

  • Закон Ома гласит, что V = I R. Таким образом, I = V / R и R = V / I.

  • Есть два типа цепей: последовательные и параллельные.

  • При последовательном соединении ток, протекающий только по одному пути, будет одинаковым при прохождении через каждый резистор.

  • При параллельном подключении напряжение на всех резисторах в цепи остается неизменным, а ток распределяется между ветвями.


Посмотрите это видео для получения дополнительной информации


Другие чтения

Комбинация резисторов — Series и Paralle

Combination of series and parallel resistors

Особенности курса

  • 101 Видеолекция
  • Примечания к редакции
  • Документы за предыдущий год
  • Ментальная карта
  • Планировщик обучения
  • Решения NCERT
  • Обсуждение Форум
  • Тестовая бумага с видео-решением

,

Электрическое сопротивление в последовательных и параллельных сетях

Последовательное соединение

series circuits

Общее сопротивление для резисторов, подключенных последовательно, можно рассчитать как

R = R 1 + R 2 + …. + R n (1)

где

R = сопротивление (Ом, Ом)

  • Резисторы — стандартные значения
Пример — Резисторы в серии

Три резистора 33 Ом , 33 Ом и 47 Ом соединены последовательно.Общее сопротивление можно рассчитать как

R = ( 33 Ом) + ( 33 Ом) + ( 47 Ом)

= 113 Ом

  • Резисторы — Цветовые коды Калькулятор

Параллельное соединение

electrical parallel circuit

Общее сопротивление для резисторов, подключенных параллельно, можно рассчитать как

1 / R = 1 / R 1 + 1 / R 2 +…. + 1 / R n (2)

Эквивалентное сопротивление двух параллельно подключенных резисторов можно выразить как

R = R 1 R 2 / (R 1 + R 2 ) (3)

Пример — параллельные резисторы

Три резистора 33 Ом , 33 Ом и 47 Ом подключены параллельно. Общее сопротивление можно рассчитать как

1 / R = 1 / ( 33 Ом ) + 1 / ( 33 Ом ) + 1 / (47 Ом )

= 0.082 (1 / Ом)

R = 1 / (0,082 Ом)

= 12,2 Ом

Если напряжение батареи 12 В — ток в цепи можно рассчитать с помощью закон

I = U / R

= (12 В) / (12,2 Ом)

= 0,98 ампер

Можно рассчитать ток через каждый резистор

I 1 = U / R 1 = (12 В) / (33 Ом) = 0.36 ампер

I 2 = U / R 2 = (12 В) / (33 Ом) = 0,36 ампер

I 3 = U / R 3 = (12 В) / (47 Ом) = 0,26 ампера

Резисторы, подключенные параллельно — Calculator

Сложите сопротивления до пяти параллельно подключенных резисторов и (необязательно) напряжение цепи.

Общее сопротивление и ток, а также отдельные токи во всех резисторах будут рассчитаны:

R 1 (Ом) I 1 (ампер)

R 2 (Ом) I 2 (ампер)

R 3 (ом) I 3 (ампер)

R 4 (ом ) I 4 (ампер)

R 5 (Ом) I 5 (ампер)

Напряжение (В)

R (Ом) I (амперы)

,Серия

и параллельные схемы — learn.sparkfun.com

Избранные любимец 48 Серия

и параллельные схемы

Простые схемы (состоящие всего из нескольких компонентов) обычно довольно просты для понимания новичками. Но когда на вечеринку приходят другие компоненты, все может стать неприятным. Куда идет ток? Что делает напряжение? Можно ли это упростить для облегчения понимания? Не бойся, бесстрашный читатель.Ценная информация следует.

В этом руководстве мы сначала обсудим разницу между последовательными и параллельными схемами, используя схемы, содержащие самые основные компоненты — резисторы и батареи, — чтобы показать разницу между двумя конфигурациями. Затем мы рассмотрим, что происходит в последовательных и параллельных цепях, когда вы комбинируете компоненты разных типов, например конденсаторы и катушки индуктивности.

Рассматривается в этом учебном пособии

  • Как выглядят конфигурации последовательной и параллельной цепей
  • Как пассивные компоненты действуют в этих конфигурациях
  • Как источник напряжения будет воздействовать на пассивные компоненты в этих конфигурациях

Рекомендуемая литература

Вы можете посетить эти учебные пособия по основным компонентам, прежде чем погрузиться в построение схем в этом учебном пособии.

Видео

Цепи серии

Узлы и текущий поток

Прежде чем мы углубимся в это, мы должны упомянуть, что такое узел . Ничего особенного, просто представление электрического соединения между двумя или более компонентами. Когда схема моделируется на схеме, эти узлы представляют собой провода между компонентами.

Пример схемы с четырьмя узлами уникального цвета.

Это полдела на пути к пониманию разницы между последовательным и параллельным. Нам также необходимо понять , как ток проходит через по цепи. Ток течет от высокого напряжения к более низкому напряжению в цепи. Некоторое количество тока будет проходить по каждому пути, который может пройти, чтобы добраться до точки с самым низким напряжением (обычно называемой землей). Используя приведенную выше схему в качестве примера, вот как будет течь ток, когда он проходит от положительной клеммы батареи к отрицательной:

Ток (обозначенный синей, оранжевой и розовой линиями), протекающий по той же схеме, что и выше.Разные токи обозначены разными цветами.

Обратите внимание, что в некоторых узлах (например, между 1 и 2 рэнд) ток на входе такой же, как на выходе. В других узлах (в частности, трехсторонний переход между R 2 , R 3 и R 4 ) основной (синий) ток разделяется на два разных. Это ключевое различие между последовательным и параллельным подключением!

Определение схем серии

Два компонента соединены последовательно, если они имеют общий узел и если через них протекает один и тот же ток .Вот пример схемы с тремя последовательными резисторами:

В указанной выше цепи есть только один способ протекания тока. Начиная с положительного полюса аккумуляторной батареи, ток сначала достигает 1 рандов. Оттуда ток будет идти прямо на 2 рандов, затем на 3 рандов и, наконец, обратно на отрицательную клемму батареи. Обратите внимание, что у текущего есть только один путь. Эти компоненты включены последовательно.

Параллельные схемы

Определение параллельных цепей

Если компоненты совместно используют два общих узла , они работают параллельно.Вот пример схемы трех резисторов, подключенных параллельно к батарее:

От положительной клеммы аккумуляторной батареи ток течет на 1 … и 2 рандов, и 3 рандов. Узел, который соединяет аккумулятор с R 1 , также подключен к другим резисторам. Другие концы этих резисторов аналогично связываются вместе, а затем снова подключаются к отрицательной клемме батареи. Существует три различных пути, по которым ток может пройти, прежде чем вернуться в батарею, и связанные резисторы считаются параллельными.

Если все последовательные компоненты имеют одинаковые токи, протекающие через них, все параллельные компоненты имеют одинаковое падение напряжения на них — series: current :: parallel: Voltage.

Совместная работа параллельных цепей и серии

Оттуда мы можем смешивать и сочетать. На следующем снимке мы снова видим три резистора и батарею. С положительной клеммы аккумуляторной батареи ток сначала достигает 1 рандов. Но на другой стороне R 1 узел разделяется, и ток может идти как на R 2 , так и на 3 R R.Затем пути тока через R 2 и R 3 снова связываются вместе, и ток возвращается к отрицательной клемме батареи.

В этом примере R 2 и R 3 идут параллельно друг другу, а R 1 идут последовательно с параллельной комбинацией R 2 и R 3 .

Расчет эквивалентных сопротивлений в последовательных цепях

Вот информация, которая может быть вам полезна.Когда мы соединяем резисторы вот так, последовательно и параллельно, мы меняем способ протекания тока через них. Например, если у нас есть питание 10 В через 10 кОм; резистора, закон Ома гласит, что у нас протекает ток 1 мА.

Если потом поставить еще 10к & ом; резистор, включенный последовательно с первым и оставив питание без изменений, мы сократили ток вдвое, потому что сопротивление увеличилось вдвое.

Другими словами, для тока по-прежнему существует только один путь, и мы только усложнили его прохождение.Насколько сложнее? 10k & Ом; + 10к & Ом; = 20 кОм ;. Вот как мы рассчитываем последовательно включенные резисторы — просто складываем их значения .

Если выразить это уравнение в более общем виде: полное сопротивление Н, — произвольное количество резисторов — это их общая сумма.

Расчет эквивалентных сопротивлений в параллельных цепях

А как насчет параллельных резисторов? Это немного сложнее, но ненамного.Рассмотрим последний пример, в котором мы начали с источника питания 10 В и 10 кОм; резистор, но на этот раз мы добавляем еще 10кОм; параллельно, а не последовательно. Теперь у тока есть два пути. Поскольку напряжение питания не изменилось, закон Ома гласит, что первый резистор по-прежнему будет потреблять 1 мА. Но то же самое и со вторым резистором, и теперь у нас есть 2 мА, исходящие от источника питания, что вдвое превышает первоначальный 1 мА. Это означает, что мы уменьшили общее сопротивление вдвое.

Пока можно сказать, что 10к & ом; || 10k & Ом; = 5 кОм; («||» примерно переводится как «параллельно»), у нас не всегда будет 2 одинаковых резистора.Что тогда?

Уравнение для добавления произвольного количества резисторов параллельно:

Если обратные значения вам не подходят, мы также можем использовать метод, называемый «произведение на сумму», когда у нас есть два резистора параллельно:

Однако этот метод подходит только для двух резисторов в одном вычислении. Используя этот метод, мы можем объединить более 2 резисторов, взяв результат R1 || R2 и вычисление этого значения параллельно с третьим резистором (снова как произведение на сумму), но обратный метод может быть меньше работы.

Время эксперимента — Часть 1

Что вам понадобится:

Давайте проведем простой эксперимент, чтобы доказать, что все работает именно так, как мы говорим.

Во-первых, мы собираемся подключить 10 кОм; последовательно подключите резисторы и наблюдайте, как они складываются самым необычным образом. Используя макетную плату, поместите один 10 кОм; резистор, как показано на рисунке, и измерьте его мультиметром. Да, мы уже знаем, что на нем будет указано 10 кОм, но это то, что мы в бизнесе называем «проверкой работоспособности».Убедившись, что мир существенно не изменился с тех пор, как мы в последний раз смотрели на него, поместите еще один аналогично, но с выводами каждого резистора, электрически подключенными через макетную плату, и измерьте снова. Измеритель теперь должен показывать что-то близкое к 20 кОм.

Вы можете заметить, что сопротивление, которое вы измеряете, может быть не совсем тем, что резистор должен быть. Резисторы имеют определенный допуск , что означает, что они могут быть отключены на определенный процент в любом направлении.Таким образом, вы можете прочитать 9.99k & ohm; или 10.01кОм. Пока оно близко к правильному значению, все должно работать нормально.

Читателю следует продолжать это упражнение до тех пор, пока он не убедится в том, что он знает, каков будет результат, прежде чем делать это снова, или у него закончатся резисторы, которые можно вставить в макет, в зависимости от того, что наступит раньше.

Время эксперимента — Часть 2

Теперь давайте попробуем это с резисторами в конфигурации параллельно .Поместите один 10 кОм; резистор в макетной плате, как и раньше (мы полагаем, что читатель уже верит, что один резистор 10 кОм будет измерять на мультиметре что-то близкое к 10 кОм). Теперь поместите второй 10k & ohm; резистор рядом с первым, следя за тем, чтобы выводы каждого резистора находились в электрически соединенных рядах. Но перед тем, как измерить комбинацию, вычислите, используя метод «произведение над суммой» или обратный метод, каким должно быть новое значение (подсказка: оно будет 5 кОм).Затем измерьте. Это что-то близкое к 5к & ом ;? Если это не так, дважды проверьте отверстия, в которые вставлены резисторы.

Повторите упражнение с 3, 4 и 5 резисторами. Расчетные / измеренные значения должны быть 3,33 кОм, 2,5 кОм; и 2кОм соответственно. Все ли получилось по плану? Если нет, вернитесь и проверьте свои соединения. Если это так, EXCELSIOR! Прежде чем продолжить, выпейте молочный коктейль. Ты заслужил это.

Практические правила для последовательных и параллельных резисторов

Есть несколько ситуаций, которые могут потребовать творческих комбинаций резисторов.Например, если мы пытаемся создать очень специфический источник опорного напряжения вы почти всегда необходимо очень специфическое соотношение резисторов, значения которых вряд ли будут «стандартные» значения. И хотя мы можем получить очень высокую степень точности номиналов резисторов, мы можем не захотеть ждать X дней, необходимых для доставки чего-либо, или платить цену за нестандартные значения, отсутствующие на складе. Так что в крайнем случае мы всегда можем создать собственные номиналы резисторов.

Совет №1: Равные резисторы, включенные параллельно

Добавление N резисторов аналогичного номинала R , включенных параллельно, дает нам R / N Ом.Допустим, нам нужен 2,5 кОм; резистор, но все, что у нас есть, это ящик, полный 10 кОм. Объединение четырех из них параллельно дает нам 10 кОм / 4 = 2,5 кОм.

Совет № 2: Допуск

Знайте, какую терпимость вы можете терпеть. Например, если вам нужен 3.2k & ohm; резистор, можно было поставить 3 10кОм; резисторы параллельно. Это даст вам 3,3 кОм, что составляет примерно 4% отклонения от необходимого значения. Но если схема, которую вы строите, должна иметь допуск ближе, чем 4%, мы можем измерить наш запас в 10 кОм, чтобы увидеть, какие из них самые низкие, потому что у них тоже есть допуск.По идее, если заначка 10к & ом; все резисторы имеют допуск 1%, мы можем получить только 3,3 кОм. Но производители запчастей, как известно, допускают именно такого рода ошибки, поэтому стоит немного покопаться.

Совет № 3: Номинальная мощность при последовательном / параллельном подключении

Такая комбинация резисторов последовательно и параллельно работает и с номинальной мощностью. Допустим, нам нужен 100 & Ом; резистор рассчитан на 2 Вт (Вт), но все, что у нас есть, это связка 1 кОм; резисторы на четверть ватта (Вт) (а сейчас 3 часа ночи, вся Mountain Dew исчезла, а кофе остыл).Вы можете объединить 10 из 1 кОм, чтобы получить 100 Ом; (1 кОм / 10 = 100 Ом), а номинальная мощность будет 10×0,25 Вт, или 2,5 Вт. Не очень красиво, но это поможет нам завершить финальный проект и может даже принести нам дополнительные баллы за способность думать на ногах.

Нам нужно быть немного более осторожными, когда мы объединяем резисторы разных номиналов параллельно, когда речь идет об общем эквивалентном сопротивлении и номинальной мощности. Для читателя это должно быть совершенно очевидно, но …

Совет №4: разные резисторы параллельно

Суммарное сопротивление двух резисторов разного номинала всегда меньше, чем резистор наименьшего номинала.Читатель будет удивлен тем, сколько раз кто-то объединяет значения в своей голове и приходит к значению, которое находится посередине между двумя резисторами (1 кОм || 10 кОм; НЕ равняется чему-либо около 5 кОм ;!). Общее параллельное сопротивление всегда будет приближаться к резистору с наименьшим значением. Сделайте себе одолжение и прочитайте совет №4 10 раз.

Совет №5: Параллельное рассеивание мощности

Мощность, рассеиваемая при параллельной комбинации резисторов разных номиналов, не распределяется между резисторами равномерно, потому что токи не равны.Используя предыдущий пример (1k & ohm; || 10k & ohm;), мы видим, что 1k & ohm; будет потреблять в 10 раз больше тока 10 кОм. Поскольку закон Ома гласит, что мощность = напряжение x ток, отсюда следует, что 1 кОм; резистор рассеивает в 10 раз мощность, превышающую 10 кОм.

В конечном счете, уроки советов 4 и 5 заключаются в том, что мы должны уделять более пристальное внимание тому, что мы делаем при параллельном соединении резисторов разных номиналов. Но советы 1 и 3 предлагают несколько удобных ярлыков, когда значения совпадают.

Конденсаторы серии

и параллельные

Объединение конденсаторов аналогично объединению резисторов … только наоборот. Как бы странно это ни звучало, это абсолютная правда. Почему это могло быть?

Конденсатор — это просто две пластины, расположенные очень близко друг к другу, и его основная функция — удерживать целую группу электронов. Чем больше значение емкости, тем больше электронов она может удерживать. Если размер пластин увеличивается, емкость увеличивается, потому что физически больше места для электронов.А если пластины раздвинуть дальше друг от друга, емкость уменьшается, потому что напряженность электрического поля между ними уменьшается с увеличением расстояния.

Теперь предположим, что у нас есть два конденсатора по 10 мкФ, соединенных последовательно, и предположим, что они оба заряжены и готовы к разрядке в друга, сидящего рядом с вами.

Помните, что в последовательной цепи есть только один путь для прохождения тока. Отсюда следует, что количество электронов, выходящих из колпачка снизу, будет таким же, как и количество электронов, выходящих из колпачка наверху.Значит, емкость не увеличилась?

На самом деле все еще хуже. Разместив конденсаторы последовательно, мы эффективно раздвинули пластины друг от друга, потому что расстояние между пластинами двух конденсаторов складывается. Так что у нас нет 20 мкФ или даже 10 мкФ. У нас 5 мкФ. Результатом этого является то, что мы добавляем значения последовательного конденсатора так же, как мы добавляем значения параллельного резистора. И метод «произведение над суммой», и метод взаимности действительны для последовательного добавления конденсаторов.

Может показаться, что нет смысла добавлять конденсаторы последовательно. Но следует отметить, что мы получили вдвое большее напряжение (или номинальное напряжение). Как и в случае с батареями, когда мы соединяем конденсаторы последовательно, напряжения складываются.

Добавление конденсаторов параллельно похоже на добавление резисторов последовательно: значения просто складываются, никаких уловок. Почему это? Их параллельное расположение эффективно увеличивает размер пластин без увеличения расстояния между ними.Чем больше площадь, тем больше емкость. Просто.

Время эксперимента — Часть 3

Что вам понадобится:

Давайте посмотрим, как работают последовательно и параллельно подключенные конденсаторы. Это будет немного сложнее, чем примеры резисторов, потому что измерить емкость напрямую мультиметром труднее.

Давайте сначала поговорим о том, что происходит, когда конденсатор заряжается с нуля вольт. Когда ток начинает идти в один из выводов, равное количество тока выходит из другого.А если последовательно с конденсатором нет сопротивления, то может быть довольно большой ток. В любом случае ток течет до тех пор, пока конденсатор не начнет заряжаться до значения приложенного напряжения, и медленнее будет стекать, пока напряжения не станут равными, когда ток полностью прекратится.

Как указано выше, потребляемый ток может быть довольно большим, если нет последовательного сопротивления с конденсатором, а время зарядки может быть очень коротким (например, миллисекунды или меньше). Для этого эксперимента мы хотим иметь возможность наблюдать за зарядкой конденсатора, поэтому мы будем использовать 10 кОм; резистор, включенный последовательно, чтобы замедлить действие до точки, где мы его легко увидим.Но сначала нам нужно поговорить о том, что такое постоянная времени RC.

В приведенном выше уравнении говорится, что одна постоянная времени в секундах (называемая тау) равна сопротивлению в омах, умноженному на емкость в фарадах. Просто? Нет? Продемонстрируем на следующей странице.

Время эксперимента — часть 3, продолжение …

В первой части этого эксперимента мы будем использовать один резистор 10 кОм и один резистор 100 мкФ (что равно 0,0001 фарад). Эти две части создают постоянную времени в 1 секунду:

При зарядке конденсатора емкостью 100 мкФ через 10 кОм; резистора, мы можем ожидать, что напряжение на цоколе вырастет примерно до 63% от напряжения питания за 1 постоянную времени, которая составляет 1 секунду.После 5 постоянных времени (в данном случае 5 секунд) конденсатор заряжается примерно на 99% до напряжения питания, и он будет следовать кривой заряда, похожей на график ниже.

Теперь, когда мы это знаем, мы собираемся подключить цепь, показанную на схеме (убедитесь, что полярность на этом конденсаторе правильная!).

С помощью нашего мультиметра, установленного для измерения вольт, проверьте выходное напряжение батареи при включенном переключателе. Это наше напряжение питания, и оно должно быть около 4.5В (будет немного больше, если батарейки новые). Теперь подключите схему, убедившись, что переключатель на аккумуляторном блоке находится в положении «ВЫКЛ», прежде чем вставлять его в макетную плату. Также позаботьтесь о том, чтобы красный и черный провода были в нужных местах. Если это более удобно, вы можете использовать зажимы типа «крокодил», чтобы прикрепить измерительные щупы к ножкам конденсатора для измерения (вы также можете немного раздвинуть эти ножки, чтобы облегчить задачу).

Когда мы убедимся, что схема выглядит правильно, а наш измеритель включен и настроен на считывание вольт, переведите переключатель на батарейном блоке в положение «ВКЛ».Примерно через 5 секунд показания счетчика должны быть довольно близкими к напряжению аккумуляторной батареи, что демонстрирует, что уравнение верное, и мы знаем, что делаем. Теперь выключите выключатель. Он все еще довольно хорошо держит это напряжение, не так ли? Это потому, что ток не может разрядить конденсатор; у нас разомкнутая цепь. Для разряда конденсатора можно использовать еще один резистор на 10 кОм параллельно. Примерно через 5 секунд он вернется к почти нулю.

Experiment Time — Часть 3, и даже больше…

Теперь мы переходим к интересным моментам, начиная с последовательного подключения двух конденсаторов. Помните, что мы сказали, что результат будет аналогичен параллельному соединению двух резисторов. Если это правда, мы можем ожидать (используя произведение над суммой)

Что это будет делать с нашей постоянной времени?

Имея это в виду, подключите другой конденсатор последовательно с первым, убедитесь, что счетчик показывает ноль вольт (или около того), и переведите переключатель в положение «ON».Зарядка до напряжения аккумуляторной батареи занимала примерно половину времени? Это потому, что емкость вдвое меньше. Электронный бензобак стал меньше, поэтому на его зарядку уходит меньше времени. Для этого эксперимента предлагается третий конденсатор, чтобы доказать это, но мы держим пари, что читатель может увидеть надпись на стене.

Теперь мы попробуем подключить конденсаторы параллельно, помня, что мы говорили ранее, что это будет похоже на добавление резисторов последовательно. Если это правда, то мы можем ожидать 200 мкФ, верно? Тогда наша постоянная времени станет

Это означает, что теперь потребуется около 10 секунд, чтобы увидеть, как параллельные конденсаторы заряжаются до напряжения питания 4.5V.

Для доказательства начнем с нашей исходной схемы на 10 кОм; резистор и один конденсатор 100 мкФ, подключенные последовательно, как показано на первой схеме этого эксперимента. Мы уже знаем, что конденсатор заряжается примерно за 5 секунд. Теперь подключите второй конденсатор параллельно. Убедитесь, что показания измерителя близки к нулю (разрядите через резистор, если он не показывает нулевое значение), и переведите переключатель на батарейном блоке в положение «ON». Нужно много времени, правда? Разумеется, мы увеличили размер электронного бензобака, и теперь он требует больше времени для его заполнения.Чтобы убедиться в этом, попробуйте добавить третий конденсатор емкостью 100 мкФ и понаблюдайте, как он заряжается в течение долгого времени.

Серия

и параллельные индукторы

Катушки индуктивности серии

и параллельные

Случаи, когда катушки индуктивности должны быть добавлены последовательно или параллельно, довольно редки, но не редкость. В любом случае, давайте рассмотрим их для полноты картины.

Вкратце, они складываются так же, как и резисторы, то есть они складываются со знаком плюс, когда включены последовательно, и с превышением произведения, когда подключены параллельно.Сложность возникает, когда они размещаются близко друг к другу, чтобы иметь взаимодействующие магнитные поля, намеренно или нет. По этой причине предпочтительнее иметь один компонент, а не два или более, хотя большинство индукторов экранированы для предотвращения взаимодействия магнитных полей.

В любом случае достаточно сказать, что они добавляют, как резисторы. Дополнительная информация о катушках индуктивности выходит далеко за рамки этого руководства.

Ресурсы и движение вперед

Теперь, когда вы знакомы с основами последовательных и параллельных схем, почему бы не ознакомиться с некоторыми из этих руководств?

  • Делители напряжения — одна из самых простых и повторяющихся схем — это делитель напряжения.Это схема, которая действительно основана на концепциях, рассмотренных в этом руководстве.
  • Что такое Ардуино? — Теперь, когда у вас есть основы схемотехники, вы можете перейти непосредственно к изучению микроконтроллеров с одной из самых популярных платформ: Arduino.
  • Основы работы с коммутатором
  • — В этом руководстве мы говорили о некоторых из основных элементов схемы, но это не был один из них. Переключатели являются важным компонентом практически в каждом электронном проекте.Узнайте все о переключателях в этом руководстве
  • Шитье проводящей нитью — схемы не обязательно должны состоять из макетов и проводов. Электронный текстиль использует токопроводящую нить для вшивания светильников и другой электроники в одежду или другую ткань.
,

Добавить комментарий

Ваш адрес email не будет опубликован. Обязательные поля помечены *